You are on page 1of 43

.

 VISIONIAS
www.visionias.in
ANSWERS & EXPLANATIONS
GENERAL STUDIES (P) TEST – 3815 (2023)

Q 1.C
x The Air sports included various sports events which are going to be in air medium. India as a developing
country has the potential to become one of the top countries in the world of air sports. India has a huge
geographical expansion and favorable weather which can be a way of promoting air sports. National Air
Sport Policy (NASP), 2022, lists various games under this category. These include aerobatics, aero
modeling and rocketry, ballooning, amateur-built and experimental aircraft, drones, gliding, and power
gliding.
x National Air Sports Policy was launched by Minister of Civil Aviation and not by the Ministry of Youth
Affairs and Sports. Hence statement 1 is not correct.
x There will be four-tier governance structure for air sports in India, Air Sports Federation of India
(ASFI) as the apex governing body. Hence statement 2 is correct.
x NASP 2022 lays out the vision of making India one of the top sports nations by 2030, by providing a
safe, affordable, accessible, enjoyable, and sustainable air sports ecosystem in India. Hence statement 3 is
correct.

Q 2.B
x India has a long history of studies on measurement of poverty. The erstwhile Planning Commission was the
nodal agency in India for estimation of poverty. Based on the methodologies suggested by the Expert
Groups/Committees set up by the Planning Commission from time to time, India has undertaken periodic
assessments of the incidence of poverty since the 1960s.
x Poverty line estimation in India has been based on the consumption expenditure and not on the
income levels due to difficulties in assessing incomes of self-employed people, daily wage laborers etc,
large fluctuations in income due to seasonal factors, additional side incomes as well as data collection
difficulties in largely rural and informal economy of India. Since households may be able to access credit
markets or household savings and thereby smooth their consumptions to some degree, consumption
expenditures may be able to provide a better basis for determining a household’s actual standard of
living. Hence, most of the Poverty Estimation Committees proposed that per capita consumption
expenditure or household expenses were the right statistical choice for calculating poverty in
India. Hence statement 1 is not correct.
x The erstwhile Planning Commission released the estimates of poverty as number of persons below poverty
line as a percentage of Indian population. In July 2013, based on the Tendulkar poverty line, Planning
Commission released poverty data for 2011-12. The number of poor in the country was pegged at 269.8
million or 21.9% of the population. After this, no official poverty estimates in India have been released.
x In 2015, Niti Ayog set up a Task Force on Poverty under the then Vice-Chairman, Niti Ayog, Prof Arvind
Panagariya. The Task Force deliberated the issue of whether a Poverty Line is required. It was stated
that Poverty line and the poverty ratio have three potential uses: identification of poor; allocation of
expenditure on anti-poverty programs across states or regions; and tracking poverty over time and across
regions. In India, identification of poor is done by the State Governments based on information from
Below Poverty Line (BPL) censuses of which the latest is the Socio-Economic Caste Census 2011
(SECC 2011). Allocation of expenditures on anti-poverty programs is also done using instruments other
than the poverty ratio. Hence statement 2 is correct.
Limitations of Poverty estimation in India:
x The use of a minimum adequate norm of nutrition as a key criterion for defining the poverty line has come
in for criticism, both for the level at which it has been fixed and for the inadequacy of the expenditure level
1 www.visionias.in ©Vision IAS
.
 of households at which these norms are likely to be met in providing a minimum standard of living. As
such, conceptual and empirical approaches to the measurement of poverty and the interpretation of data are
not universally accepted.
 x Discussion on determining the nutritional or calorie norm in which the poverty line is rooted has been long
and complex. Poverty line should vary over regions mainly because of the variations of the tastes and
preferences and the price structures over the regions. Hence, determining components of Poverty Line
Basket (PLB) has been one of the key challenge of poverty line estimation in India due to price differentials
(of constituents of basket) which vary from state to state and period to period. Hence statement 3 is correct.

Q 3.D
x XR (Extended Reality) is the collective term applied to immersive experiences incorporating varying
degrees and real information.
x AR (Augmented Reality): User views static digital information or visual elements virtual elements
integrated into the real environment.
x MR (Mixed Reality): User interacts with responsive virtual elements integrated into the real environment.
x VR (Virtual Reality): User is immersed in an interactive, digitally-generated environment.

x Hence option (d) is the correct answer.

Q 4.B
x The decline of the Pratihara dynasty in the 10th century CE opened the way for their own feudatory chiefs
to declare themselves as independent powers. The Chalukyas, Chandellas, Chahmanas, Gahadawalas,
Paramaras, Kalachuris, and Guhilas, all of whom were the feudatory chiefs of the Gurjara-Pratiharas in
different regions, thus, became independent as distinct Rajput clans in their own territories.
x The Kalachuris- Kalachuris, who were in the service of the Gurjara-Pratiharas as feudatories, also
did not hesitate to declare their independence. They were also known as Kalachuris of Chedi or
Tripuri. Hence option (b) is the correct answer.
o They ruled the Chedi region from their capital Tripuri (modern Tewar near Jabalpur, Madhya Pradesh).
o Gangeyadeva was one of the most important rulers of Kalachuri dynasty as his policies aimed to make
the Chedis the paramount power of North India.
x The Gahadawalas occupied Kannauj in the 11th century. From Kannauj they ruled over the major
portions of the Gangetic doab during 1090-1193. The Gahadawala king Jayachandra is usually styled as the
king of Benares by Muslim historians owing to his intimate connection with the city of Banaras.
x Chahamanas came into prominence after the decline of the Gurjara-Pratiharas. There were numerous
branches of the Chahamanas but some of them were unquestionably the feudatories of Pratiharas of Avanti
and Kannauj. It also remains a fact that during 750-950 CE most of the regions ruled by the Chahamanas
formed part of Pratihara dominion. In 973 CE they became practically independent. The city of Ajayameru
(modern Ajmer) founded by king Ajayaraja was their political center and seat of power.
x The Chandellas ruled over central India between the 10th-13th centuries CE. Their territory was known as
Jejakabhukti (modern Bundelkhand).
x Another contemporary Rajput political power – the Paramaras – emerged in the region of Gujarat,
Malwa, and Southern Rajputana out of the bitter struggle between the Gurjara-Pratiharas and the

2 www.visionias.in ©Vision IAS


.
 Rashtrakutas. The territory ruled over by the Paramaras of Malwa included Malwa proper and the adjoining
districts.
x The Kachhapagatas- The Kachhapagatas were at first the feudatories of the Gurjara-Pratiharas. They
 made themselves masters of the fort of Gwalior after defeating the ruler of Kannauj. During the 10th and
11th centuries, the region of eastern Rajaputana and Gwalior residency was overruled by three independent
branches of Kacchapagatas.

Q 5.C
x It is necessary for a candidate to make and subscribe to an oath or affirmation before an officer
authorised by the Election Commission. For any particular election, the authorised persons are,
principally, the Returning Officer and the Assistant Returning Officer for the constituency. Hence,
statement 2 is correct.
x In the case of a candidate confined in prison or under preventive detention, the superintendent of the prison
or commandant of the detention camp in which he is so confined or is under such detention is authorised to
administer the oath.
x And in the case of a candidate confined to bed in a hospital or elsewhere owing to illness or any other cause,
the medical superintendent in charge of the hospital or the medical practitioner attending to him is similarly
authorised.
x If a candidate is outside India, the Indian Ambassador or the High Commissioner or diplomatic consular
authorised by him can also administer an oath/affirmation.
x Returning Officer (RO)
o The Returning Officer of a Parliamentary or assembly constituency is responsible for the conduct
of elections in the Parliamentary or assembly constituency concerned. Hence, statement 1 is
correct.
o The Election Commission of India nominates or designates an officer of the Government or a local
authority as the Returning Officer for each assembly and parliamentary constituency in consultation
with the State Government / Union Territory Administration.
o In addition, the Election Commission of India also appoints one or more Assistant Returning Officers
for each of the assembly and Parliamentary constituencies to assist the Returning Officer in the
performance of his functions in connection with the conduct of elections.
x Presiding Officer
o The Presiding Officer with the assistance of polling officers conducts the poll at a polling station.
o The District Election Officer appoints the Presiding Officers and the Polling Officers.
o In the case of Union Territories, such appointments are made by the Returning Officers. Hence,
statement 3 is correct.

Q 6.C
x Seigniorage is the difference between the value of money and the cost to produce it — in other words, it's
the economic cost of producing a currency within a given economy or country. If the seigniorage is positive,
then the government will make an economic profit; a negative seigniorage will result in an economic loss.
x If, for example, it costs the Indian government 5 rupees to produce a note of Rs 100, the seigniorage is 95
rupees or the difference between the two amounts. Seigniorage gives a country the potential to turn a profit
when it produces money.
x Hence option (c) is the correct answer.

Q 7.A
x Recently RBI issued a framework for operations of subsidiaries and branches of Indian banks and All India
Financial Institutions (AIFIs) in foreign jurisdictions and in International Financial Services Centers
(IFSCs) in India, including Gujarat International Finance Tec-City (GIFT City).
x As per the framework, the foreign branches/foreign subsidiaries of Indian banks/AIFIs can deal in financial
products, including structured financial products, which are not available or are not permitted by the Reserve
Bank in the domestic market without prior approval of the Reserve Bank.
x The branches/subsidiaries of Indian banks/AIFIs operating in IFSCs including those operating out
of GIFT City may also deal in financial products, including structured financial products, which are
not available or are not permitted by the Reserve Bank in the domestic market subject to compliance
with all applicable laws/regulations. Hence statement 1 is correct.
x Specifying conditions for dealing in financial products, the RBI said the banks and AIFIs will have to ensure
that such branches and subsidiaries do not deal in products linked to Indian Rupee unless specifically
permitted by Reserve Bank. They cannot also accept structured deposits from any Indian resident.
3 www.visionias.in ©Vision IAS
.
 x Further, the financial products dealt with by the foreign branches and subsidiaries as well as IFSCs
will attract prudential norms such as capital adequacy, exposure norms, periodical valuation, and all
other applicable norms.
 x RBI also said the activities of branches/subsidiaries in foreign jurisdictions and IFSCs would be
subject to the laws in India unless specifically exempted by law. Hence statement 2 is not correct.

Q 8.A
x The W boson is a fundamental particle. It has a significantly greater mass than theorized by the
Standard Model. Hence statement 2 is not correct.
x Together with the Z boson, it is responsible for the weak force, one of the four fundamental particles that
govern the behaviour of matter in our universe. Particles of matter
can interact by exchanging these bosons, but only over short
distances.
x With a charge of +1 or -1, the W boson can change the identity of
particles.
x It is responsible for many nuclear processes, including those
letting stars burn. This burning also creates heavier elements and,
when a star dies, those elements are tossed into space as the
building blocks for planets and people. Hence statement 1 is
correct.
x The Standard Model:
o It is a set of mathematical formulae and measurements
describing elementary or fundamental particles and their
interactions.
o It contains 12 fundamental matter particles categorized as
quarks and leptons and three forces that govern the behavior of matter: electromagnetism, the strong
and weak nuclear forces.
ƒ The force of gravity is currently not included in the standard model.

Q 9.D
x Recently, the United States and 60 other partner countries signed a political declaration called the
"Declaration for the Future of the Internet". The document is an agreement to prevent digital
authoritarianism.
x The countries that have signed the declaration include the US, European Union, United Kingdom,
Canada, France, Austria, Belgium, Bulgaria, Cabo Verde, Canada, Colombia, Costa Rica, Croatia,
Cyprus, Czech Republic, Denmark, Dominican Republic, Estonia, the European Commission, and
Finland. India, China, and Russia are among the large nations that are not part of this declaration.
India also did not sign the Budapest Convention on Cybercrime, 2001.
x The Declaration's principles include commitments to protect human rights and fundamental freedoms of all
people; promote a global Internet that advances the free flow of information; advance inclusive and
affordable connectivity so that all people can benefit from the digital economy; promote trust in the global
digital ecosystem, including through protection of privacy; and protect and strengthen the multistakeholder
approach to governance that keeps the Internet running for the benefit of all.
x A total of 182 internet crackdowns were reported globally in 2021. Out of 106 shutdowns in India, 85 were
reported in Jammu and Kashmir. India was one of 18 countries that blocked mobile internet during protests.
The number of countries that shut down the internet in 2021 has increased to 34 from 29 in 2020. The
Declaration and its guiding principles are not legally binding. It should be used as a reference point for
public policymakers, as well as citizens, businesses, and Civil Society Organisations.
x Hence option (d) is the correct answer.

Q 10.D
x Lord Curzon was appointed as the viceroy of India in 1899 in succession of lord Elgin II. There was not a
single department where the administrative reforms of Curzon were not introduced. Curzon aimed at the
efficiency of the administration and sought to strengthen "officialization and centralization. Taking no
account of the feelings and aspirations of the Indian people, Curzon wanted to establish the stronghold of
the British Empire. Curzon after becoming a viceroy appointed an expert Commission to probe into the
working of department and then enacted necessary legislation.
x In 1902, Curzon appointed a Police commission under the President ship of sir Andrew Frazer. The
police commission appointed by Curzon was entrusted with the task of enquiring the efficient
4 www.visionias.in ©Vision IAS
.
 functioning of the police administration of every province. The reports of the commission published in
the year of 1903. According to the reports the police force was far from being efficient, defective in training
and organization, inadequately supervised and oppressive. The Commission recommended the increase of
 salary of the police thereby strengthening the efficiency of the police department in all provinces. Moreover
the commissions sought the constructions of the training schools for the officers and the constables, direct
recruitments in the place of promotion in the s higher ranks. According to the reports of the Commission
a provincial police service was created. Central Department of Criminal intelligence was also
recommended by the Commission, which would enquire the criminal cases both in towns and the villages.
Most of the recommendations of the commissions were accepted and implemented by Curzon. During the
reign of Curzon there was an increase of expenditure on the Police department. Hence statement 1 is
correct.
x The existing system of education in India however proved faulty to Curzon. According to Curzon
educational institutions in India had become the production house of the political revolutionaries. Thus the
growth of the political revolutionaries in the schools would be a great threat to the British supremacy in
India. Hence Curzon took the educational reforms as one of the significant part of his administrative
reforms.
o In 1902, the university commission was appointed to enquire into the condition of s Universities in
India and to recommend proposal for the improvement of the education procedure in the Universities.
o On the basis of the Reports approached by the Commission, the Indian universities Act (1904) was
passed.
ƒ The Act sought to increase the official control over the universities. In doing this, the number of
fellows from the administrative body of the universities was limited, which increased the nominated
elements over the elected fellows. Moreover the power of veto rested on the government.
ƒ Hence only the government can only prohibit the regulation passed by the Senates. Conditions for
affiliation of the private colleges were made more rigid and the periodical inspection by the
syndicate was made compulsory. The universities were forced to take active part in the promotion
of study and research.
x The administrative reforms of Curzon also included the economic reforms. Curzon was shrewd diplomat
and he could well understood that unless he controlled the financial set up properly, the company would
lose its administrative hold in India. Curzon passed legislations regarding the famines, land revenues,
Irrigation, Agriculture, Railways, taxation and currency. The famine and drought of 1899 had affected
wide areas in the north, south central and western India. Hence a famine Commission was appointed
under the President ship MacDonnell. The Commission was entrusted to enquire into the results of
the famine operations. According to the reports of the Commissions the relief distributed to the famine
stricken people was excessive. The commission recommended the payment by physical work by the able
bodied person and laid down several rules to deal with the fodder famine. Hence statement 2 is not
correct.
x In 1901, a Commission was appointed under the Chairmanship of Sir Colin Scott Moncrieff. This
commission was entrusted with the department of irrigation. The commission recommended an
additional expenditure 4 ½ crores of rupees on irrigation. The construction work on the Jhelum canal
was completed and other works were taken into hand in order to develop the irrigation works. Hence
statement 3 is not correct.
x Besides for the improvement of the Indian agriculture and the livestock, scientific methods of cultivation
were adopted. Moreover an Imperial Agriculture Department was set up under the direct supervision
of the Inspector general. Hence statement 4 is correct.
x Curzon in order to extend and strengthen his control on the trade and commerce of the country constituted
a new Department of Commerce and industry. . This department was entitled to look after the entire
industrial and the commercial interest in India. This department looked after the Posts and telegraphs,
factories, Railway Administration, Mines, Ports, and Marine etc. the Indian coinage and the paper Currency
Act of 1899 made a British sovereign legal tender in India at the rate of Rs. 15 to a sovereign. Thus India
was put on a gold standard.

Q 11.C
x The ruins of the imperial capital of Vijayanagara are spread over 25 kilometres in the rocky outcrops of
Hampi.The Royal Centre was located in the south-western part of the settlement. Although designated as a
Royal Centre, it included over 60 temples, indicating that the patronage of temples and cults was important
for rulers to establish and legitimise their authority through association with the divinities housed in the
shrines.

5 www.visionias.in ©Vision IAS


.
 x The Vijayanagara ruler, Devaraya I (1406–1422), constructed the famous Hazara Rama Temple as a
shrine to Ramachandra in the fifteenth century. Situated in the middle of the Royal Centre, it functioned
as a state temple and was used by the Vijayanagara rulers and their private family members.
 x Since the Hazara Rama temple was a royal temple, the frescoes and sculptures on the walls of the temple
depicted royal authority and power. The reliefs inside the temple on various portions illustrate scenes from
the Ramayana and are a representation of the artistic excellence of the artisans and sculptors present in the
imperial city.
x Immediately south of the main entrance to the Hazara Rama temple is a sequence of two gateways leading
to the two important structures of the Royal Centre. These structures are the Audience Hall and Great
Platform. The audience hall comprises 100 stone footings, probably props for the wooden pillars that
disappeared long ago.
x Nearby is the multi-storied Great Platform, popularly associated with the Mahanavami festival. Constructed
in the fourteenth century, the Platform, or Mahanavami dibba has three ascending and diminishing stages,
each a solid square, added at a different time. Steps lead up the platform from the south and west sides, but
there is no structure on top, only the recently exposed stone footings of a vanished wooden pavilion.
x The platform is often identified with the House of Victory. On the basis of the accounts of the foreign
travellers who visited the city, the king is supposed to have witnessed the celebrations of the Mahanavami
festival from the top of the platform, which had a temporary shrine dedicated to Durga. Relief carvings at
the bottom stage of the platform depict diverse images of royal life. There is a stepped tank immediately to
the south, and other nearby bathing places were probably used on particular festival occasions.
x Short distances from the Mahanavami dibba are two structures whose architectural layouts are influenced
by Islamic styles. One is the Queen’s Bath, and the other is the Lotus Mahal. The queens’ bath was
probably intended for the amusement of the Vijayanagara king and his courtiers. It has an ornate interior
arcade with balconies running around a sunken square pool.
x The two-storied Lotus Mahal was probably a royal pavilion. Like the queens’ bath, it is built in the
fanciful Vijayanagara courtly style, influenced by Islamic designs and motifs. It is located in the middle of
a high-walled compound. A vaulted hall nearby may have served as a treasury or gymnasium. Octagonal
and square watchtowers with balconies are located outside. A small doorway to the east leads to the elephant
stables, suggesting that the Lotus Mahal enclosure was an abode of men rather than a zenana, or women’s
quarter, as is sometimes believed.
x The Aihole Durga temple is not related to the Vijayanagara period. The temple has been dated to the
late 7th and early 8th centuries of the early Chalukya dynasty. The hybridisation and incorporation of
several styles was the hallmark of Chalukyan buildings.
x The Durga temple at Aihole is unique, having an earlier style of an apsidal shrine that is reminiscent of
Buddhist chaitya halls and is surrounded by a veranda of a later kind, with a shikhara that is stylistically
like a Nagara one.
x Hence option (c) is the correct answer.
Q 12.C
x The Partition of India in 1947 was the division of British India into two independent dominion states, the
Union of India and the Dominion of Pakistan. The division of British India between the two new dominions
was carried out according to what has come to be known as the “Mountbatten Plan”. It was announced at a
press conference by Mountbatten on 3 June 1947, when the date of independence – 15 August 1947 – was
also announced.
x The partition of India was accompanied by division of resources of the civil government as well as division
of the military forces and equipment.To resolve the division of civil government amicably, a partition
council, presided over by the governor-general and consisting of two representatives each of India and
Pakistan, was set up.
x Before the Partition Council, a Partition Committee was formed which was chaired by Lord Mountbatten
and its members were Vallabh Bhai Patel, Dr. Rajendra Prasad, Liaqat Ali Khan, and Abdur Rab Nishtar.
x Later this committee was replaced by a Partition Council. In this council, Congress was represented
by Sardar Patel and Dr. Rajendra Prasad, with C. Rajgopalachari as an alternate member. Muslim league
was represented by Mohammed Ali Jinnah, Liaqat Ali Khan, and Abdur Rab Nishtar as an alternate
member. The main job of this Partition Council was to oversee the division of assets and personnel between
India and Pakistan. Hence option (c) is the correct answer.
o Nishtar and C. Rajagopalachari were to participate in the council meetingsif and when one of the two
regular members representing Pakistan and India was unable to attend them.
o An Arbitral Tribunal was also set up anticipating disputes between the two sides.
x The council was helped by a steering committee, consisting of H.M. Patel and Mohammad Ali, at
operational level.
6 www.visionias.in ©Vision IAS
.
 Q 13.A
x RIMPAC (Rim of the Pacific Exercise) is the world's largest international maritime Multilateral Naval
Exercise held biennial by the USA. 27 countries are participating in the current edition of the multi-
 dimensional exercise. Hence statement 1 is correct and statement 2 is not correct.
x The exercise is intense operations and training aimed at enhancing interoperability and building trust among
Navies of friendly foreign countries.
x In 1971, the inaugural RIMPAC brought together military personnel from Australia, Canada, New
Zealand, the United Kingdom (UK), and the United States (US). Since then, Australia, Canada, and the
US have consistently taken part in all subsequent RIMPAC exercises. In 2014, India joined RIMPAC for
the first time by deploying the INS Sahyadri, a domestically produced Shivalik class stealth frigate, to
participate in the exercise.

Q 14.B
x Recent context: The black hole at the center of the Milky Way galaxy was photographed for the first
time.
x Black hole is a place in space where gravity pulls so much that even light cannot get out.
o This can happen when a star is dying.
o Presence of black holes can be inferred by detecting their effect on other matter nearby them.
o Importance of studying Black Holes: for testing fundamental theories, understanding of gravitational
force etc.
x Basic parts of Black Hole:
o Schwarzschild Radius: This is the event horizon's radius. It is the radius at which the escape velocity
is equal to the speed of light.
o Ergosphere: If the black hole is rotating, then as it spins, itsmass causes the space time around the
black hole to rotate as well. This region is called the ergosphere.

x Hence option (b) is the correct answer.


x Types of Black Holes:

o Tiny Black Holes:


ƒ These can be as small as one atom but have mass of a large mountain.
ƒ Formed when universe began.
o Stellar Black Holes:
ƒ Mass up to 20 times more than mass of sun.
ƒ Formed when centre of a very big star falls upon itself or collapses.
ƒ When this happens, it causes a supernova.
o Supermassive Black Holes:
ƒ Mass more than 1 million suns together.
ƒ Formed at the same time as the galaxy they are in.
Q 15.D
x The new Foreign Contribution (Regulation) Amendment Rules, 2022, were notified by the ministry of
home affairs (MHA) through a gazette notification.
x Foreign Contribution means the donation, delivery, or transfer made by any foreign source of any
article, currency, or security.
o Now individuals can send up to Rs. 10 lakhs without informing the government.
o If the amount exceeds Rs. 10 lakhs, the individuals will have three months to inform the
government against 30 days earlier.
7 www.visionias.in ©Vision IAS
.
 o The list of compoundable offenses was increased to 12 from 7.
ƒ Compoundable offenses are those offenses where, the complainant enters into a compromise, and
agrees to have the charges dropped against the accused. However, such a compromise should be a
 "Bonafide," and not for any consideration to which the complainant is not entitled.
o Organizations receiving foreign funds will not be able to use more than 20 % of such funds for
administrative purposes. This limit was 50 % before 2020.
x With the Ministry of Home Affairs affecting changes in the Foreign Contribution (Regulation) Act
(FCRA) and its rules through two gazette notifications in 2022, political parties, legislature members,
election candidates, judges, government servants, journalists, and media houses among others — all
barred from receiving foreign contribution – will no longer be prosecuted if they receive foreign
contribution from relatives abroad and fail to intimate the government within 90 days. The
government orders can be challenged in the High Courts.
x Hence, option (d) is the correct answer.

Q 16.C
x The Mahatma Gandhi National Rural Employment Guarantee Act 2005 (MGNREGA) aims at enhancing
the livelihood security of households in rural areas of the country by providing at least 100 days of
guaranteed wage employment in a financial year to every household whose adult members volunteer to do
unskilled manual work. In case employment is not provided within 15 days from the date of registration of
the demand for work or the date from which work has been demanded in case of advance applications,
whichever is later, the worker is entitled to a daily unemployment allowance.
x Achievements under MGNREGA:
o Geo-tagging of the Assets: Geo MGNREGA Phase-I was rolled out on 1 September 2016 for
geotagging of all completed works. The geo-tagging of assets is carried out in three stages viz: before
initiation of work, during the work, and after completion of work.
o Mandatory expenditure on agriculture and allied activities: As per the provision of the Act, the
District Programme Coordinator is required to ensure that at least 60 percent of the works to be
taken up in a district in terms of cost shall be for the creation of productive assets directly linked
to agriculture and allied activities through development of land, water, and trees. The expenditure
on agriculture and allied activities is 68.5 percent in FY23. Hence statement 3 is correct.
o e-Payments: e-payments are done under MGNREGA using National Electronic Fund Management
System (Ne-FMS)/ Electronic Fund Management System (e-FMS) for the payment of wages of workers
directly to their bank/post office accounts. So far, total expenditure through NeFMS/e-FMS is 99.7
percent.
o DBT: Under the scheme, 99 percent of wage seekers are receiving their wages directly into their
bank accounts. It is a big step towards transparency. Hence statement 1 is correct.
x A total of 5.6 crore households availed employment and a total of 225.8 crore person-days employment has
been generated under the Scheme (until 6 January 2023). The physical progress of MGNREGS in terms of
person-days generation, average person-days per household, and participation of women are indicated
below.

x From the above, Women


participation rate is more than fifty percent. Hence statement 2 is not correct.

Q 17.D
x Two recent sets of data released by the National Sample Survey Office (NSSO) and the National Statistical
Office (NSO) offer insights into the process of structural transformation in the Indian economy, especially
in relation to the agriculture and manufacturing sectors.
x The NSSO’s latest annual Periodic Labour Force Survey (PLFS) report for 2021-22 (July-June) shows the
farm sector’s share in the country’s employed labour force at 45.5%. That’s down from 46.5% in 2020-21,
but still higher than the 2018-19 low of 42.5%. Clearly, the effects of the pandemic-induced economic
disruptions, which had forced a reverse migration to the farms, haven’t fully subsided.

8 www.visionias.in ©Vision IAS


.
 x The share of agriculture in the total workforce fell from 64.6% in 1993-94 to 42.5% in 2018-19. The biggest
decline, from 58.5% to 48.9%, happened between 2004-05 and 2011-12. The share of manufacturing (and
mining) in total employment has actually fallen along with that of agriculture. The share of the labour force
 employed in manufacturing too peaked at 12.6% in 2011-12.

x From the above table, the share of the workforce in manufacturing has decreased over the last
decade. Hence statement 1 is not correct.
x From the above table, it can be seen that the employment share of the construction sector has neither steadily
increased nor steadily decreased. Hence statement 2 is not correct.

Q 18.C
x Recently, China has successfully cloned a wild Arctic wolf for the first time in the world. Hence
statement 2 is correct.
x Cloning:
o Clone: Copied material, which has the same genetic makeup as the original, is referred to as a clone.
o Cloning: Process of producing genetically identical copies of a biological entity (genes, cells, tissues,
and even entire organisms), either by natural or artificial means.
o Not identical: Despite having the same genetic material clones do not always look identical as
environment also plays a role in deciding the physical feature of an organism.
o In mammals including humans, identical twins (monozygotic twins) are natural clones. Hence
statement 1 is correct.
ƒ These twins are produced when a fertilized egg splits, creating two or more embryos that carry
almost identical genetic material.
o Applications of Cloning:
ƒ Cloned embryo could produce stem cells.
ƒ Help in studying diseases.
ƒ Recreating extinct species.
ƒ More milk or meat in livestock

9 www.visionias.in ©Vision IAS


.
 Q 19.A
x Green Methanol:
o It is methanol that is produced renewably and without polluting emissions, one of its variants being
 generated from green hydrogen. It is a low-carbon fuel that can be made from either biomass
gasification or renewable electricity and captured carbon dioxide (CO2). Hence statement 1 is
correct.
o This chemical compound can be used as a low-carbon liquid fuel and it is a promising alternative to
fossil fuels in areas where decarbonization is a major challenge, such as maritime transport.
Recently, State-owned power generation company NTPC has partnered with Tecnimont to
explore green methanol production at a commercial scale. The green methanol project involves
capturing carbon from NTPC power plants and converting it into green fuel. Hence statement 2
is correct.
x Methanol (CH3OH) is an alcohol that can be used as an alternative fuel in the form of a blend with
petroleum fuel or on itself. Methanol is not to be mistaken for ethanol (C2H5OH), which can also be
used as an alternative fuel.
o Methanol fuel is mostly produced by the catalytic oxidation of methane from natural gas to form
methanol, ( Which can be replaced by Green Technologies) which is then converted to dimethyl ether
(DME), which can be used as a diesel fuel.
o Methanol fuel is used in internal combustion engines in pure form or in a mixture with gasoline
or diesel.
o Pure methanol is used in many different race car competitions due to its increased thermal efficiency,
low cost, and low risk of flammability. It also does not create opaque smoke when on fire, unlike
diesel or gasoline.
o The combustion of pure methanol has no nitrogen oxide (NOx) emissions, no sulfur oxide
(SOx) emissions and very low particulate matter (PM) and carbon dioxide (CO2) emissions
compared to gasoline or diesel. Hence statement 3 is not correct.
o A mixture of methanol and petroleum fuel will result in lower emissions than regular petroleum fuel.
Methanol fuel is toxic, but it does not evaporate as quickly as petroleum fuel and it also can be
extinguished with water.

Q 20.B
x Recent Context: Mission Innovation (MI) launched Integrated Bio-refineries Mission (IBM) to accelerate
Clean Energy solutions through Public-Private Alliances.
x Mission Innovation is a global initiative to catalyze action and investment in research, development and
demonstration to make clean energy affordable, attractive and accessible to all this decade. Hence
statement 1 is not correct.
o It consists of 22 countries and European Commission (on behalf of European Union).
o India is a founding member.
o First phase of the mission was launched alongside the Paris Agreement in 2015 and 2nd phase of
MI, was launched in 2021. Hence statement 2 is correct.
x About IBM:
o It is launched with the goal of replacing 10% of fossil-based fuels, chemicals, and materials with bio-
based alternatives by 2030.
o This is the 6th Mission launched by Mission Innovation.
o Other 5 missions includes Clean Hydrogen, Green Powered Future, Zero-Emission Shipping, Carbon
Dioxide Removal, and Urban Transitions.
Q 21.A
x Audit of Accounts of Co-operative Societies: The state legislature may make provisions for the
maintenance of accounts by the cooperative societies and the auditing of such accounts at least once in each
financial year. Hence, statement 1 is not correct.
o It shall lay down the minimum qualifications and experience of auditors and auditing firms that
shall be eligible for auditing the accounts of the cooperative societies. Hence, statement 2 is not
correct.
o Every cooperative society shall be audited by an auditor or auditing firm, appointed by the general body
of the cooperative society. But, such an auditor or auditing firm shall be appointed from a panel
approved by the State Government or a body authorised by the State Government on this behalf.
o The accounts of every cooperative society shall be audited within six months of the close of the
financial year. Hence, statement 3 is correct.
o The audit report of the accounts of an apex cooperative society shall be laid before the state legislature.

 10 www.visionias.in ©Vision IAS


.
 Q 22.B
x The West African monsoon and the Indian monsoon are two separate monsoon systems that are not directly
related to each other. However, they are both affected by the same global climate patterns, such as the
 El Niño-Southern Oscillation (ENSO), the Indian Ocean Dipole (IOD), and the Atlantic Multi-Decadal
Oscillation (AMO), which can influence their strength and timing.
x The West African monsoon occurs during the summer months (June-September) and is characterized
by a shift in the prevailing wind direction from the northeast to the southwest, bringing moisture from the
Atlantic Ocean to the Sahel region of West Africa. The Indian monsoon, on the other hand, occurs
during the summer months (June-September) and is characterized by a shift in the prevailing wind
direction from the southwest to the northeast, bringing moisture from the Indian Ocean to the Indian
subcontinent. Hence statement 1 is not correct.
x The strength and timing of the West African monsoon and the Indian monsoon are influenced by a variety
of factors, including sea surface temperature anomalies, atmospheric pressure patterns, and land surface
conditions. For example, a warmer-than-normal Atlantic Ocean and cooler-than-normal Indian Ocean can
enhance the West African monsoon and weaken the Indian monsoon, while a cooler-than-normal Atlantic
Ocean and warmer-than-normal Indian Ocean can have the opposite effect. Hence statement 2 is correct.
Q 23.C
x IUCN World Heritage Outlook 3 builds on three cycles of Conservation Outlook Assessments undertaken
since 2014(2014, 2017 and 2020). It serves as an indicator of the effectiveness of protected and conserved
areas at a time when the international community seeks to measure progress towards global biodiversity
targets and defines the Post-2020 Global Biodiversity Framework.
o Focusing on the natural values for which sites are inscribed, threats to these values, and the effectiveness
of actions to protect them, the IUCN World Heritage Outlook assesses the conservation prospects
of all natural World Heritage sites. Hence statements 1 and 2 are correct.
o These sites are globally recognized as the most significant natural areas on Earth and their conservation
must meet the high standards of the World Heritage Convention. Our ability to conserve these sites is
thus a litmus test for the broader success of conservation worldwide.
x The results of the IUCN World Heritage Outlook 3 indicate, that
o 63% of all sites (159) the conservation outlook is either “good” or “good with some concerns”,
o while for 30% (75 sites) the outlook is of “significant concern”,
o and for 7% (18 sites) it is assessed as “critical”.
x The conservation outlook for natural World Heritage sites in 2020 is similar to the overall results in 2017,
with a 1% decrease in sites assessed as either “good” or “good with some concerns” between 2017 and
2020. It shows that conservation prospects remain positive for almost two thirds of all natural sites, while
also indicating that further significant efforts are required to improve the outlook of many sites.

Q 24.C
x Recently, SC has observed that Doctrine of Res Judicata is attracted not only in separate subsequent
proceedings but also a subsequent stage of same proceedings.
x Res Judicata is a phrase which has been evolved from a Latin maxim, which stand for ‘the thing has been
judged', meaning there by that the issue before the court has already been decided by another court, between
the same parties.
o Thus it prevents a party from bringing a claim once that particular claim has been subjected to
a final judgment in some previous lawsuit. Hence statement 1 is correct.
o Therefore, the court will dismiss the case before it as being useless. Res Judicata as a concept is
applicable both in case of civil as well as criminal legal system.
x It provides finality to litigation and protects parties from being vexed by same matter twice.
x It is defined under Section 11 of the Code of Civil Procedure, 1908. Hence statement 2 is correct.
x Doctrine of Res Sub Judice bars two parallel suits between the same parties and is applicable to pending
proceedings.

Q 25.A
x Electromagnetic waves are waves that can travel through a vacuum. Electromagnetic radiation is a form
of radiant energy released by certain electromagnetic processes.
o Visible light is one type of electromagnetic radiation;
o other familiar forms are invisible electromagnetic radiation such as X-rays and infrared radiation etc.
x A sound is a form of a mechanical wave that needs a physical medium for its propagation. Therefore,
ultrasonic waves (high-frequency sound waves) are not examples of electromagnetic waves.
x Hence option (a) is the correct answer.
 11 www.visionias.in ©Vision IAS
.
 Q 26.D
x The Carbon Border Adjustment Mechanism (CBAM) is a policy tool being developed by the European
Union (EU) to address the issue of carbon leakage, which occurs when production shifts to countries with
 lower environmental standards or weaker climate policies. The CBAM aims to level the playing field for
EU industries that are subject to high carbon costs, by placing a price on imported goods based on their
carbon footprint. Hence statement 1 is not correct.
x Under the CBAM, importers would have to buy permits equivalent to the carbon emissions that are
produced in the production of their imported goods. This would create a financial incentive for non-EU
countries to reduce their carbon emissions in line with the EU's climate goals, as they would otherwise face
higher costs when exporting their goods to the EU.
x A group of countries including India has opposed the carbon border taxes policy at the COP27 in
Sharm El Sheikh, saying it could “result in market distortion”. Hence statement 2 is not correct.
x The BASIC group, comprising India, China, Brazil and South Africa, said in a joint statement,
“Unilateral measures and discriminatory practices, such as carbon border taxes, that could result in market
distortion and aggravate the trust deficit amongst Parties, must be avoided. BASIC countries call for a united
solidarity response by developing countries to any unfair shifting of responsibilities from developed to
developing countries.”

Q 27.D
x The Shadow zone is the zone in the earth’s interior from where earthquake waves are not reported.
Shadow zones occur due to the bending of earthquake waves as they pass from one medium to another.
o P-wave shadow zone: P-waves are not detected between 105-145 degrees from the focus which is
called the P-wave shadow zone. It is due to the bending and slowing down(due to change in density) of
P-waves as it enters the outer core. Hence statement 1 is not correct.
o S-wave shadow zone: S-waves cannot pass through the outer core which is liquid and it results in an
S-wave shadow zone beyond 105 degrees from the focus. Hence statement 2 is not correct.
x Significance of shadow zones:
o The earthquake waves undergo reflection, bending, or changes in direction while passing from one
medium to another.
o The "outer core" was discovered when it was found that P-waves were bent inwards thereby producing
a "shadow zone" at the surface.
o Since the S-waves do not pass through the outer core, it is concluded that they may be in a liquid state.
o We know that the inner core is solid by identifying a “phase-shift” of seismic waves that travel through
it.
o By measuring the time it takes for seismic waves to travel along many different paths through the earth,
we can figure out the velocity structure of the earth.
x The travel profile of the seismic waves not only indicates the position of each layer but also gives clues as
to its composition. The information is utilized for infrastructural and mining projects and also for damage
mitigation.

Q 28.D
x In August 1921, peasant discontent erupted in the Malabar district of Kerala. Here Moplah (Muslim)
tenants rebelled. Their grievances were related to lack of any security of tenure, renewal fees, high rents,
and other oppressive Hindu landlord exactions. Hence statement 1 is correct.
o The Mappilas had expressed their resentment against the oppression of the landlords (mainly
Hindus) during the nineteenth century also.
 12 www.visionias.in ©Vision IAS
.
 x Their grievances centred around lack of security of tenure, high rents, renewal fees and other oppressive
exactions.
x The Mappila tenants were particularly encouraged by the demand of the local Congress body for a
 government legislation regulating tenant-landlord relations.
x Soon, the Mappila movement merged with the ongoing Khilafat agitation. Hence statement 3 is
correct.
x The leaders of the Khilafat-Non-Cooperation Movement like Gandhi, Shaukat Ali and Maulana Azad
addressed Mappila meetings. Hence statement 2 is correct.
x After the arrest of national leaders, the leadership passed into the hands of local Mappila leaders.
x Things took a turn for the worse in August 1921 when the arrest of a respected priest leader, Ali
Musaliar, sparked off large-scale riots. Initially, the symbols of British authority—courts, police stations,
treasuries and offices— and unpopular landlords (jenmies who were mostly Hindus) were the targets.
x But once the British declared martial law and repression began in earnest, the character of the rebellion
underwent a definite change.
x Many Hindus were seen by the Mappilas to be helping the authorities. What began as an
antigovernment and anti-landlord affair acquired communal overtones.
x The communalisation of the rebellion completed the isolation of the Mappilas from the Khilafat-
NonCooperation Movement.
x By December 1921, all resistance had come to a stop.

Q 29.B
x Fearing mass action by the Congress, a Congress-dominated Interim Government headed by Nehru
was sworn in on September 2, 1946 with Nehru continuing to insist on his party’s opposition to the
compulsory grouping.
x Despite the title, the Interim Government was little more than a continuation of the old executive of the
viceroy (Wavell overruled the ministers on the issue of the release of INA prisoners in his very last
cabinet meeting in March 1947).
x Wavell quietly brought the Muslim League into the Interim Government on October 26, 1946.
x The League was allowed to join
o without giving up the ‘direct action’;
o despite its rejection of the Cabinet Mission’s longterm and short-term plans; and
o despite insistence on compulsory grouping with decisions being taken by a majority vote by a section
as a whole (which would reduce the opponents of Pakistan in Assam and NWFP to a position of helpless
minority).
x 14 Ministers of Interim Government (September 2, 1946–August 15, 1947)
o Jawaharlal Nehru: Vice President of Executive Council, External Affairs and Common Wealth
Relations
o Vallabhbhai Patel: Home, Information and Broadcasting
o Baldev Singh: Defence
o Dr. John Mathai: Industries and Supplies
o C. Rajagopalachari: Education
o C.H. Bhabha: Works, Mines and Power
o Rajendra Prasad: Agriculture and Food
o Jagjivan Ram: Labour
o Asaf Ali: Railway
o Liaquat Ali Khan (Muslim League): Finance
o Ibrahim Ismail Chundrigar (Muslim League): Commerce
o Abdur Rab Nishtar (Muslim League): Communications
o Ghazanfar Ali Khan (Muslim League): Health
o Jogendra Nath Mandal (Muslim League): Law
x Hence, option (b) is the correct answer.
Q 30.A
x Agni is a nuclear capable ballistic missile, which is developed by the Defence Research and
Development Organisation (DRDO). The Agni series of missiles is a family of medium to long-range
nuclear-capable missiles that are designed to provide India with strategic deterrence against potential
adversaries.
x The Agni missile series includes Agni-I, Agni-II, Agni-III, Agni-IV, and Agni-V, with each missile
having its own range and capabilities. These missiles are developed indigenously in India and have been
successfully tested in a range of conditions.
 13 www.visionias.in ©Vision IAS
.
 x Agni Prime is a new, upgraded version, is a two-stage canisterised solid propellant ballistic missile with
dual redundant navigation and guidance system, successfully tested the new generation nuclear capable
ballistic missile from Dr APJ Abdul Kalam island off the coast of Odisha.
 x Hence option (a) is the correct answer.

Q 31.D
x Teaser loans are products that offer a lower rate of interest on loans for a fixed period of time, after which
the rate of interest adjusts to the market levels. Credit card, adjustable-rate mortgages and gold loans are
common products that can be counted under the teaser loan category in India. Hence option (d) is the
correct answer.
x Banks and non-banking finance companies offer competitive teaser loans in accordance with their credit
disbursement targets. To beat the rising competition and retain high-value customers, they launched teaser
loans. The withdrawal of teaser loans is expected to give a boost to the near-term net interest margin (NIM).
x While such teaser products are not banned by the regulator, the standard asset provisioning requirement is
higher for such loans. For normal home loans, the standard asset provisioning is 0.4% but for teaser loans,
it is 2%. RBI had increased the provisioning by five times for such loans since these loans are
perceived as riskier. Higher provisioning discourages banks from offering such products. Following the
introduction of higher risk weights, banks discontinued those products.

Q 32.C
x First Factories Act: The first factories Act was passed in 1881. The Act was primarily designed to
protect children and to provide some facilities for their safety at work. Since the Act of 1881 did not have
sufficient provisions for the protection of children and the regulation of female labour, the Factories
(Amendment) Act, 1891 was passed. The Act restricted the employment of women at night, raised the lower
and upper age limits for child workers in factories to 9 and 14 years respectively and reduced the daily hours
of work for children from 9 to 7 during the day time. For the first time in India, it provided for the grant. A
regular rest interval of half an hour, in the middle of the day, and a weekly day of rest to all workers in
factories.
x All India Trade Union Congress: is the oldest trade union federations in India. It was founded on 31
October 1920 with Lala Lajpat Rai as its first president in Bombay by Lala Lajpat Rai, Joseph
Baptista, N. M. Joshi, Diwan Chaman Lall and a few others. AITUC is a founder member of the World
Federation of Trade Unions.
x Trade Unions Act 1926: The objective of this Act was to provide for the registration of trade unions and
define the law relating to registered trade in certain aspects.
x The Federation of Indian Chambers of Commerce and Industry (FICCI): is an association of business
organizations in India. Established in 1927, on the advice of Mahatma Gandhi by GD Birla and
Purshottamdas Thakurdas, it is the largest, oldest and apex business organization in India. It is a non-
government, not-for-profit organization. FICCI draws its membership from the corporate sector, both
private and public, including SMEs and MNCs. The chamber has an indirect membership of over 250,000
companies from various regional chambers of commerce. It is involved in sector-specific business building,
business promotion and networking. It is headquartered in the national capital New Delhi and has a
presence in 12 states in India and 8 countries across the world. The correct sequence is 1-3-2-4.
x Hence option (c) is the correct answer.

Q 33.D
x Vijayanagara, or "city of victory" was the name of both a city and an empire. The empire was founded in
the fourteenth century. In its heyday, it stretched from the river Krishna in the north to the extreme south of
the peninsula.
x The first dynasty, known as the Sangama dynasty, exercised control until 1485. They were supplanted by
the Saluvas, military commanders, who remained in power until 1503 when they were replaced by the
Tuluvas.
x Krishnadeva Raya belonged to the Tuluva dynasty. Krishnadeva Raya’s rule was characterised by
expansion and consolidation. This was the time when the land between the Tungabhadra and Krishna rivers
(the Raichur doab) was acquired (1512), the rulers of Orissa were subdued (1514) and severe defeats were
inflicted on the Sultan of Bijapur (1520).
x Strain began to show within the imperial structure following Krishnadeva Raya’s death in 1529. His
successors were troubled by rebellious nayakas, or military chiefs.
o By 1542, control at the centre had shifted to another ruling lineage, that of the Aravidu, which remained
in power till the end of the seventeenth century. During this period, as indeed earlier, the military
 14 www.visionias.in ©Vision IAS
.
 ambitions of the rulers of Vijayanagara as well as those of the Deccan Sultanates resulted in shifting
alignments. Eventually, this led to an alliance of the Sultanates against Vijayanagara.
x In 1565, Rama Raya, the chief minister of Vijayanagara, led the army into battle at Rakshasi-Tangadi
 (also known as Talikota), where his forces were routed by the combined armies of Bijapur, Ahmadnagar
and Golconda.
o The victorious armies sacked the city of Vijayanagara. The city was totally abandoned within a few
years.
o The focus of the empire shifted to the east, where the Aravidu dynasty ruled from Penukonda and later
from Chandragiri (near Tirupati).
x Hence option (d) is the correct answer.

Q 34.A
x India's electronics industry is valued at US$118 billion and further aims to reach US$300 billion worth
of electronics manufacturing and US$ 120 billion in exports by FY26, supported by the vision of a US$ 1
trillion digital economy by 2025. Hence, statement 3 is not correct.
x The major drivers of growth in this industry are mobile phones, consumer electronics, and industrial
electronics. In the mobile phone segment, India has become the second-largest mobile phone
manufacturer globally, with the production of handsets going up from six crore units in FY15 to 31 crore
units in FY22. Hence, statement 1 is correct.
x India possesses 20 percent of the world’s semiconductor design engineers, however, only has a
minuscule share in intellectual property (IP). Hence, statement 2 is not correct.

Q 35.D
x The ECOsystem Spaceborne Thermal Radiometer Experiment on Space Station (ECOSTRESS), is
an ongoing scientific experiment of NASA in which a radiometer is mounted on the International Space
Station (ISS). Hence statement 2 is not correct.
o It will monitor one of the most basic processes in living plants: the loss of water through the tiny
pores in leaves. When people lose water through their pores, the process is called sweating. The related
process in plants is known as transpiration.
o Because water that evaporates from the soil around plants also affects the amount of water that plants
can use, ECOSTRESS will measure combined evaporation and transpiration, known
as evapotranspiration (ET). Hence statement 1 is not correct.
o ECOSTRESS will offer clues about how Earth’s water and carbon cycles affect plant growth and how
ecosystems adapt to changes in climate by measuring evapotranspiration, or the loss of water from
leaves and soil.
x ECOSTRESS will address 3 science questions:
o How is the terrestrial biosphere responding to changes in water availability?
o How do changes in diurnal vegetation water stress impact the global carbon cycle?
o Can agricultural vulnerability be reduced through advanced monitoring of agricultural water
consumptive use and improved drought estimation?

Q 36.B
x Bile is an alkaline greenish-yellow fluid and contains salts which help to emulsify or break the fats
molecules into small droplets. It makes the acidic food coming from the stomach alkaline so that
pancreatic enzymes can act on it. Hence statement 2 is correct.
x Bile juice from the liver accomplishes this in addition to acting on fats. The small intestine is the site of
the complete digestion of carbohydrates, proteins, and fats. It receives the secretions of the liver and
pancreas for this purpose. Hence statement 1 is not correct.
x Fats are present in the intestine in the form of large globules which makes it difficult for enzymes to
act on them. Bile salts break them down into smaller globules increasing the efficiency of enzyme
action.
x The pancreas secretes pancreatic juice which contains enzymes like trypsin for digesting proteins and lipase
for breaking down emulsified fats.

Q 37.A
x Charged Expenditure: The budget consists of two types of expenditure–the expenditure ‘charged’ upon
the Consolidated Fund of India and the expenditure ‘made’ from the Consolidated Fund of India. The
charged expenditure is non-votable by the Parliament, that is, it can only be discussed by the Parliament,
while the other type has to be voted by the Parliament. The list of the charged expenditure is as follows:
 15 www.visionias.in ©Vision IAS
.
 o Emoluments and allowances of the President and other expenditures relating to his office.
o Salaries and allowances of the Chairman and the Deputy Chairman of the Rajya Sabha and the Speaker
and the Deputy Speaker of the Lok Sabha.
 o Salaries, allowances and pensions of the judges of the Supreme Court.
o Pensions of the judges of high courts.
o Salary, allowances and pension of the Comptroller and Auditor General of India.
o Salaries, allowances and pension of the chairman and members of the Union Public Service
Commission.
o Administrative expenses of the Supreme Court, the office of the Comptroller and Auditor General
of India and the Union Public Service Commission including the salaries, allowances and pensions
of the persons serving in these offices. Hence, option 3 is correct.
o The debt charges for which the Government of India is liable, including interest, sinking fund charges
and redemption charges and other expenditures relating to the raising of loans and the service and
redemption of debt. Hence option 1 is correct.
o Any sum required to satisfy any judgement, decree or award of any court or arbitral tribunal.
Hence option 2 is correct.
o Any other expenditure declared by the Parliament to be so charged.

Q 38.B
x The financial sector comprises commercial banks, insurance companies, non-banking financial companies,
co-operatives, pension funds, mutual funds and other smaller financial entities.
x Non-financial debt consists of credit instruments issued by governmental entities, households and
businesses that are not included in the financial sector. Non-financial debt includes household or
commercial loans, Treasury bills and credit card balances.
x Non-financial private sector debt refers collectively to debt of non-financial corporations and
households, ie the non-financial sector excluding the general government. Household debt is the stock
of liabilities held by the sector Households and Non-Profit institutions serving households.
x Hence option (b) is the correct answer.

Q 39.A
x Buddhist and Jaina texts list 16 powerful states (solasa-mahajanapada) that flourished in the early 6th
century BCE.
x The Anguttara Nikaya’s list of the mahajanapadas is as follows: Kasi (Kashi), Kosala (Koshala), Anga,
Magadha, Vajji (Vrijji), Malla, Chetiya (Chedi), Vamsa (Vatsa), Kuru, Panchala, Machchha (Matsya),
Shurasena, Assaka (Ashmaka), Avanti, Gandhara, and Kamboja. The Mahavastu has a similar list, but
substitutes Shibi (in the Punjab) and Dasharna (in central India) for the north-western states of Gandhara
and Kamboja.

x Hence option (a) is the correct answer.


 16 www.visionias.in ©Vision IAS
.
 Q 40.C
x The position of the council vis-a-vis the assembly is much weaker than the position of the Rajya Sabha vis-
a-vis the Lok Sabha.
 x The position of the Rajya Sabha and Legislative Council in cases of ordinary bills.
o Rajya Sabha has equal powers with Lok Sabha with respect to the introduction and passage of
ordinary bills.
o Legislative Council with respect to the introduction and passage of ordinary bills has equal powers to
Legislative Assemble. But, in case of disagreement between the two Houses, the will of the assembly
prevails over that of the council. Therefore, the council is subordinate to the assembly in all respects.
Thus, the predominance of the assembly over the council is fully established.
o The final power of passing an ordinary bill also lies with the assembly. At the most, the council can
detain or delay the bill for the period of four months–three months in the first instance and one month
in the second instance. In other words, the council is not even a revising body like the Rajya Sabha;
it is only a dilatory chamber or an advisory body. So, the Legislative council in the state legislature
is less powerful than the Rajya Sabha in the Parliament. Hence, statement 1 is correct.
x The position of the Rajya Sabha and Legislative Council in cases of money bills.
o A Money Bill can be introduced only in the lower house (Lok Sabha or Legislative Assembly) and not
in the upper house (Rajya Sabha or Legislative Council).
o Rajya Sabha/Council cannot amend or reject a Money Bill. It should return the bill to the Lok
Sabha/Assembly within 14 days, either with recommendations or without recommendations.
o The Lok Sabha/Assembly can either accept or reject all or any of the recommendations of the Rajya
Sabha/Council. In both cases, the money bill is deemed to have been passed by the two Houses. Hence
statement 2 is correct.
o The final power to decide whether a particular bill is a money bill or not is vested in the Speaker of the
Lok Sabha/Assembly.
x Even though both the council and the Rajya Sabha are second chambers, the Constitution has given
the council much lesser importance than the Rajya Sabha.

Q 41.A
x Nutrient cycling is of two types. gaseous and sedimentary. Atmosphere or hydrosphere is the reservoir
for the gaseous type of cycle (carbon), whereas Earth’s crust is the reservoir for the sedimentary type
(phosphorus). Hence, statement 1 is correct.
x The carbon cycle includes the uptake of carbon dioxide by plants through, its ingestion by animals and its
release to the atmosphere through respiration and decay of organic materials. Human activities like the
burning of fossil fuels contribute to the release of carbon dioxide in the atmosphere.
x The phosphorus cycle involves the uptake of phosphorus by organisms. Phosphorus in the environment is
mainly found in rocks, and natural weathering processes can make it available to biological systems. After
the decomposition of biological waste, it can accumulate in large amounts in soils and sediments.
Phosphorus is used by humans as a fertilizer in farmlands and in detergents. Overuse of phosphorus can
lead to eutrophication.
x The other two major and important differences between the carbon and phosphorus cycle are
first, atmospheric inputs of phosphorus through rainfall are much smaller than carbon inputs, and,
secondly, gaseous exchanges of phosphorus between organisms and the environment are negligible. Hence,
statement 2 is not correct.

Q 42.B
x A person charged under sedition law is barred from a government job. • They have to live without their
passport and must produce themselves in court at all times as and when required.
x Indian Penal Code (IPC) defines sedition (Section 124A) as an offence committed when any person by
words or otherwise brings or attempts to bring into hatred or contempt, or excites or attempts to excite
disaffection towards the government established by law in India by
o words, either spoken or written
o signs
o visible representation, or otherwise
x Sedition is a cognisable, nonbailable and non-compoundable offence under the law. Hence statement
1 is correct.
x A person charged under sedition law is barred from a government job.
o They have to live without their passport and must produce themselves in court at all times as and
when required. Hence statement 2 is correct.
 17 www.visionias.in ©Vision IAS
.
 x In 2018, Law Commission of India (LCI) recommended to repeal section 124A of IPC. Hence
statement 3 is not correct.
x History of sedition law
 o In 1837, British historian-politician Thomas Macaulay drafted sedition law as an offence punishable
with life imprisonment.
o First case registered, when Jogendra Chandra Bose, editor of newspaper "Bangobasi" was booked for
publishing an article criticizing "Age of consent Bill.
o
o Indira Gandhi government made Section 124A cognisable offence that authorises police to make arrests
without a warrant.
o In Kedar Nath Verdict, Constitution bench uphold the validity of sedition law.
o Kedar Nath Vs State of Bihar, 1962: SC also held that citizen have right to criticize or comment on
government, or its actions, as long as he does not provoke violence.
o P. Alavi vs State of Kerala, 1982: SC held that sloganeering, criticising of Parliament or Judicial setup
did not amount to sedition.

Q 43.B
x The Prime Minister is appointed by the President, while the other ministers are appointed by the President
on the advice of the Prime Minister. This means that the President can appoint only those persons as
ministers who are recommended by the Prime minister.
x Usually, the members of Parliament, either Lok Sabha or Rajya Sabha, are appointed as ministers. A person
who is not a member of either House of Parliament can also be appointed as a minister. But, within six
months, he must become a member (either by election or by nomination) of either House of Parliament,
otherwise, he ceases to be a minister.
x A minister who is a member of one House of Parliament has the right to speak and to take part in the
proceedings of the other House also, but he can vote only in the House of which he is a member.
Therefore, a PM being a full-time member of the Rajya Sabha can speak in Lok Sabha during the
discussion of the budget. Hence, statement 3 is not correct.
x Also, he will not be able to vote when a no-confidence motion is under consideration because a no-
confidence motion is only in Lok sabha and the PM from Rajya Sabha cannot vote in Lok Sabha.
Hence, statement 1 is not correct.
x The Constitution has not fixed the term of office of members of the Rajya Sabha and left it to the
Parliament. Accordingly, the Parliament in the Representation of the People Act (1951) provided that
the term of office of a member of the Rajya Sabha shall be six years. The act also empowered the
president of India to curtail the term of members chosen in the first Rajya Sabha.
x Unlike the Rajya Sabha, the Lok Sabha is not a continuing chamber. Its normal term is five years from
the date of its first meeting after the general elections, after which it automatically dissolves. However,
the President is authorised to dissolve the Lok Sabha at any time even before the completion of five years
and this cannot be challenged in a court of law.
x Therefore, the PM being a full-term member of the Rajya Sabha will have a term of six years in Rajya
Sabha against a maximum five-year term in Lok Sabha. Hence, statement 2 is correct.

Q 44.C
x Land reforms is an integral part of the scheme of agricultural development and rural reconstruction since
the inception of economic planning in the country. As put in the words of the Second Five Year Plan, the
objective of land reform was to create conditions for evolving as speedily as possible the agrarian economy
with high levels of efficiency and productivity and to establish an egalitarian society and eliminate social
inequalities. The same objective was repeated in Fifth Five Year Plan Draft.
x The Land Reforms in India aimed at the redistribution of ownership holdings and reorganizing operational
holdings from the viewpoint of optimum utilization of land. It has also aimed at providing security of tenure,
fixation of rents, and conferment of ownership.
x The major objectives of land reform package, as identified in the Eighth Plan, are:
o Restructuring of agrarian relations to achieve an egalitarian structure
o Elimination of exploitation in land relations
o Actualization of the goal of “land to the tiller”
o Improvement of socio-economic conditions of the rural poor by widening their land base
o Increasing agricultural production and productivity
o Facilitating land-based development of rural poor
o Infusion of a great measure of equality in local institutions

 18 www.visionias.in ©Vision IAS


.
 x After Independence, the immediate goal of the Government was to increase foodgrains production by
switching over from cash crops to food crops. Therefore, Land reforms were not focused on cash crops
and hence cultivation of cash crops being the predominant form of cultivation is not an objective of
 land reforms in independent India. Hence option (c) is the correct answer.

Q 45.D
x An orbit is the curved path that an object in space (such as a star, planet, moon, asteroid or spacecraft) takes
around another object due to gravity.
x Geostationary orbit (GEO):
o Satellites in geostationary orbit (GEO) circle Earth above the equator from west to east following
Earth’s rotation – taking 23 hours 56 minutes and 4 seconds – by travelling at exactly the same rate
as Earth. This makes satellites in GEO appear to be ‘stationary’ over a fixed position. In order to
perfectly match Earth’s rotation, the speed of GEO satellites should be about 3 km per second at an
altitude of 35 786 km.
o GEO is used by satellites that need to stay constantly above one particular place over Earth, such
as telecommunication satellites.
x Low Earth orbit (LEO):
o A low Earth orbit (LEO) is, as the name suggests, an orbit that is relatively close to Earth’s surface.
It is normally at an altitude of less than 1000 km but could be as low as 160 km above Earth – which is
low compared to other orbits, but still very far above Earth’s surface.
o Unlike satellites in GEO that must always orbit along Earth’s equator, LEO satellites do not
always have to follow a particular path around Earth in the same way – their plane can be
tilted. This means there are more available routes for satellites in LEO, which is one of the reasons why
LEO is a very commonly used orbit. Hence statement 3 is correct.
o LEO’s close proximity to Earth makes it useful for several reasons. It is the orbit most commonly used
for satellite imaging, as being near the surface allows it to take images of higher resolution.
o It is also the orbit used for the International Space Station (ISS), as it is easier for astronauts to travel to
and from it at a shorter distance. Satellites in this orbit travel at a speed of around 7.8 km per second; at
this speed, a satellite takes approximately 90 minutes to circle Earth, meaning the ISS travels around
Earth about 16 times a day.
x Medium Earth orbit (MEO):
o Medium Earth orbit comprises a wide range of orbits anywhere between LEO and GEO. It is similar to
LEO in that it also does not need to take specific paths around Earth, and it is used by a variety of
satellites with many different applications.
o It is very commonly used by navigation satellites.
x Polar orbit:
o Satellites in polar orbits usually travel past Earth from north to south rather than from west to east,
passing roughly over Earth's poles.
o Satellites in a polar orbit do not have to pass the North and South Pole precisely; even a deviation
within 20 to 30 degrees is still classed as a polar orbit.Polar orbits are a type of low Earth orbit, as they
are at low altitudes between 200 to 1000 km. Hence statement 1 is correct.
x Sun-synchronous orbit (SSO):
o It is a particular kind of polar orbit. Satellites in SSO, travelling over the polar regions, are
synchronous with the Sun. This means they are synchronised to always be in the same ‘fixed’ position
relative to the Sun. This means that the satellite always visits the same spot at the same local time – for
example, passing the city of Delhi every day at noon exactly.
o Often, satellites in SSO are synchronised so that they are in constant dawn or dusk – this is
because by constantly riding a sunset or sunrise, they will never have the Sun at an angle where
the Earth shadows them. Hence statement 2 is correct.

Q 46.A
x Chandra Prabha wildlife sanctuary is located between Chakiya and Naugarh in the Chandauli district of
Uttar Pradesh. The area was made a hunting preserve for the rulers of Benares in the second half of the 18th
century. The sanctuary was set up in 1957, covering the reserved forest area in Chandraprabha and some
parts of the Jaimohini Range.
o Asiatic lions brought from the Gir forests of Gujarat were introduced in Chandra Prabha wildlife
sanctuary in 1958. However, this experiment failed badly due to the mysterious disappearance of
lions. Hence option (a) is the correct answer.

 19 www.visionias.in ©Vision IAS


.
 x Mahavir Swami Sanctuary lies amidst the thick woodlands in Lalitpur of Uttar Pradesh. It is about 125
km from Jhansi. It was set up in 1977. Mahavir Swami Sanctuary features a wide variety of wild and
endangered animals. Wildlife found here includes Neelgai, Sambhar, Leopard, Wild Bear, and lots of
 birds.
x Suhaildev/Suhelwa wildlife Sanctuary Located in the Shravasti, Balrampur, and Gonda districts of Uttar
Pradesh, Suhelwa was declared a Wildlife Sanctuary in 1988. Occupying an area of 452 sq km, the sanctuary
is covered with Sal, Sheesham, Khair, Sagaun (Teak), Asna, Jamun, Haldu, Phaldu, Dhamina, Jhingan, and
Bahera trees. The fauna found in the sanctuary includes Leopard, Tiger, Bear, Wildcat, Wild Boar, and
various birds.
x The Katarniaghat Wildlife Sanctuary is a protected area in the Upper Gangetic plain, near Bahraich city
in the Bahraich district of Uttar Pradesh, India, and covers an area of 400.6 km² in the Terai of the Bahraich
district.

Q 47.D
x The important phase in the development of music during the Sultanate period belongs to the time of Amir
Khusrau. It is during this period that the qawwali style is said to have developed. He is also credited for
the development of many modern ragas like aiman, gora and sanam. He is credited with the creation of
a new musical instrument, the sitar which was a combination of the Indian vina and the Iranian tambura.
The Turks are credited with bringing musical instruments like rabab and sarangi into South Asia. Hence,
statement 1 is not correct.
x In Vrindavan, Swami Haridas promoted music and is considered to have taught Tansen who was at the
court of the Mughal emperor Akbar. Tansen is regarded as an important exponent of Hindustani classical
music and is credited with introducing ragas as the Miyan ki Malhar, Miyan ki Todi and Darbari. Hence,
statement 2 is correct.
x Raja Mansingh is said to have played an important role in the perfection of the Dhrupad style of North
Indian Music. In the south, a system of ragas known as the Janaka and Janya ragas existed during this
period. The Swaramela Kalanidhi by Ramamatya of Kondavidu written in 1550 describes 20 Janan and 64
Janya ragas. By the 18th century, several new forms of music like Tarana, Dadra, and Ghazal had come into
existence. Hence, statement 3 is correct.

Q 48.D
x Directive Principles of State Policy have been provided under Part IV of the Indian Constitution (Article 36
- Article 51). Article 40 directs the state to take steps to organize village panchayats and endow them with
such powers and authority as may be necessary to enable them to function as units of self-government. It
does not say anything about urban local bodies (or municipalities). Hence statement 1 is not correct.
x The Constitution (74th Amendment) Act, 1992 provides that the urban areas would be governed by one of
the three mentioned types of urban local governments namely the Nagar Panchayat, Municipal Council, and
Municipal Corporation based on the size of the area. Nagar Panchayats were to administer areas in transition
from rural to urban areas, municipal councils to administer small urban areas, and municipal corporations
for larger urban areas. Article 243Q of the Indian Constitution also provides for the constitution of the
following municipalities:
o a Nagar Panchayat (by whatever name called) for a transitional area, that is to say, an area in
transition from a rural area to an urban area; Hence statement 2 is correct.
o a Municipal Council for a smaller urban area; and
o a Municipal Corporation for a larger urban area
x The Twelfth Schedule of the Indian Constitution provides for the powers, authority, and responsibilities
of Municipalities. This schedule was added by the 74th Amendment Act of 1992. It has 18 matters or
subjects. And one such subject is cattle pounds and the prevention of cruelty to animals. Hence statement
3 is correct.

Q 49.D
x Ramdeo Misra is revered as the Father of Ecology in India. Born on 26 August 1908, Ramdeo Misra
obtained Ph.D. in Ecology (1937) under Prof. W. H. Pearsall, FRS, from Leeds University in the UK. He
established teaching and research in ecology at the Department of Botany of the Banaras Hindu University,
Varanasi. Hence option (d) is the correct answer.
x His research laid the foundations for understanding tropical communities and their succession,
environmental responses of plant populations, and productivity and nutrient cycling in tropical forest and
grassland ecosystems. Misra formulated the first postgraduate course in ecology in India. Over 50 scholars

 20 www.visionias.in ©Vision IAS


.
 obtained Ph. D degrees under his supervision and moved on to other universities and research institutes to
initiate ecology teaching and research across the country.
x He was honored with the Fellowships of the Indian National Science Academy and World Academy of Arts
 and Science, and the prestigious Sanjay Gandhi Award in Environment and Ecology. Due to his efforts,
the Government of India established the National Committee for Environmental Planning and
Coordination (1972) which, in later years, paved the way for the establishment of the Ministry of
Environment and Forests (1984).
x M. S. Swaminathan or Monkombu Sambasivan Swaminathan: He was born on August 7, 1925, in
Kumbakonam, Tamil Nadu, India. He is an agricultural scientist, plant geneticist, Indian agronomist,
administrator, and humanitarian. He is renowned for his leading role in India's "Green Revolution." It is a
programme under which high-yield varieties of wheat and rice seedlings are planted in the fields of poor
farmers.
x Salim Ali or Salim Moizuddin Abdul Ali: He was born on November 12, 1896, in the Bombay Presidency,
British India. He was also known as the "Birdman of India." He was an Indian ornithologist and naturalist.
He was also the first Indian who conducted systematic bird surveys across India and wrote various bird
books that popularised ornithology in India. He played an instrumental role in creating the Bharatpur Bird
Sanctuary (Keoladeo National Park).
x Sunderlal Bahuguna: He was born on January 9, 1927, near Tehri, Uttarakhand, British India. He was an
Indian environmentalist and the leader of the Chipko movement. He fought for the preservation of forests
in the Himalayas. In 1970, he first fought as a member of the Chipko movement and later spearheaded the
anti-Tehri Dam movement from the 1980s to early 2004.

Q 50.A
x Ministry of Power and New & Renewable Energy, has initiated the ‘#iCommit’ campaign, on the
occasion of World Environment Day (5th June). The initiative is a clarion call to all stakeholders and
individuals to continue moving towards energy efficiency, renewable energy, and sustainability to create a
robust and resilient energy system in the future. Hence option (a) is the correct answer.
x The ‘#iCommit’ initiative, driven by Energy Efficiency Services Limited (EESL), under the
administration of the Ministry of Power, Government of India is uniting a diverse set of players such as
Governments, Corporates, Multilateral and Bilateral Organisations, Think Tanks and Individuals.
x The ‘#iCommit’ initiative is centered around the idea of building an energy-resilient future. The prerequisite
for that goal is to create a flexible and agile power system. A healthy power sector can help the nation in
meeting the objective of energy access and security for all.
x With the imminent changes in the power system, brought about by innovations such as decentralised solar
and electric vehicles, a collaboration between all stakeholders will be the way forward and is at the core of
‘#iCommit’ campaign.

Q 51.B
x Recently, retired SC judge Ranjana Prakash Desai became the first women chairperson of Press Council of
India (PCI).
x It is a Statutory, quasi-judicial authority. It was set up under the PCI Act, 1978. Hence statement 1
is not correct.
x Tenure of chair person is three years.
o Chairman is selected by the LS Speaker, RS Chairman and a member elected by the PCI. Hence
statement 2 is correct.
x Composition
o Chairman, who has by convention, been a retired SC judge, and 28 members.
o Five members from Parliament, three nominated by LS Speaker and two nominated by Chairman of
RS.
x Functions
o Watchdog of press, for press and by press.
o Adjudicates complaints against and by press for violation of ethics and freedom of press
respectively. Hence statement 3 is correct.

Q 52.A
x The terms r-selection and K-selection are used by ecologists to describe the growth and reproduction
strategies of organisms. Hence option (a) is the correct answer.
o r-selected species have a high growth rate but low survivability (“cheap” offspring)
o K-selected species have a low growth rate but high survivability (“expensive” offspring)

 21 www.visionias.in ©Vision IAS


.
 x r-selection:
o Occurs in unstable environments where there are ecological disruptions and resources are used for
maximizing reproduction
 o There are usually many offspring per brood, which require little parental care and have a high rate of
mortality
o The body size of offspring is typically small and they have an early onset of maturity (short
developmental span)
o Population size is typically variable (highly fluctuating) and an example of an r-selected organism is a
pioneer species
x K-selection:
o Predominates in stable or predictable environments where resources are used for maximizing long-
term survival
o There are usually very few offspring per brood, each requiring high levels of parental care (resulting
in low mortality)
o The body size of offspring is typically larger and they have a late onset of maturity (long
developmental span)
o Population size is typically stable (reaches carrying capacity) and an example of K-selection is a climax
species.
o The r-K Scale of Reproductive Strategy: Offspring Numbers

Q 53.D
x Monetary policy essentially deals with the supply and cost (interest rates) of money in an economy. The
RBI’s MPC meets every two months to assess the state of monetary activities, and may tweak the repo rate
— the interest rate at which the RBI lends to commercial banks — in a manner that reduces price
fluctuations in the economy while keeping the inflation rate (the rate at which the general price level in the
economy grows) at a reasonable level.
x There are two aspects to any monetary policy: the decision on repo rate, and the “stance” of the policy.
While changes to the repo rate affect the broader economy in terms of whether loans — for homes, cars, or
factories — will be costlier or cheaper, the policy stance opens a window into how members of the MPC
see the inflation and economic growth situation. The panel indicates its broader policy approach by
guiding the markets with policy stances. MPC's decision on the repo rate will have an immediate
effect on interest rates in the economy. But it is not the case with the stance as it is just indicative in
nature.
x Various stances of RBI
o An accommodative stance means the central bank is prepared to expand the money supply to boost
economic growth. The central bank, during an accommodative policy period, is willing to cut interest
rates. A rate hike is ruled out. The central bank typically adopts an accommodative policy when growth
needs policy support and inflation is not the immediate concern.
o A ‘neutral stance’ suggests that the central bank can either cut rate or increase rate. This stance is
typically adopted when the policy priority is equal on both inflation and growth. The interest rate can
move to either sides depending on incoming data.
o A hawkish stance indicates that the central bank expects high inflation in the economy and its top
priority is to keep inflation low. During such a phase, the central bank is willing to hike interest rates to
curb the money supply and thus reduce the demand. A hawkish policy also indicates tight monetary
policy. A rate cut is nearly certain during such a period.
x The central bank typically gives policy guidance to financial markets (through its stance) so that all types
of investors can make informed decisions. The MPC is not bound to follow its stance, meaning if the policy
 22 www.visionias.in ©Vision IAS
.
 stance to markets is accommodative, it is not mandatory on part of the MPC to cut rates in the future. Future
actions of the MPC will be dependent on incoming data. If one looks at the stated policy stance and actual
actions by the central bank in the last four years, out of fourteen monetary policy actions, where it has stated
 ‘accommodative’ stance, the RBI has cut rates on six occasions and left rates untouched on eight
occasions. Hence if RBI maintained the policy stance at withdrawal of accommodation it may or may
not lead to an increase in the interest rates in the future.
x Recent MPC stance: Since retail inflation has been outside RBI’s comfort zone of 2% to 6% for 10 of the
past 12 months, the monetary policy committee has continued to retain its 'Withdrawal of Accommodation'
stance to bring down high inflation, especially the core inflation. This may be considered a hawkish stance
as RBI is more concerned about high inflation than about the moderation in economic growth. Hence
option (d) is the correct answer.

Q 54.B
x The Aegean sea is situated between the continents of Europe and Asia that extends from the
Mediterranean Sea. It is positioned between the regions of the Balkans and Anatolia. Hence statement 1
is not correct.
x The Aegean Sea is linked to the Marmara Sea and the Black Sea via the Dardanelles and Bosphorus
straits. The sea is home to numerous Aegean Islands, including Crete and Rhodes, which are situated along
its southern edge.
x Island of Crete is considered as the southern limit of Aegean sea. Hence statement 2 is correct.

Q 55.C
x The Ministry of Jal Shakti and the Ministry of Housing and Urban Affairs have jointly launched
the River Cities Alliance (RCA). Hence statement 2 is not correct.
x It is a dedicated platform for cities in India that are situated along rivers.
x The RCA will focus on three broad themes: Networking, Capacity Building, and Technical Support, and
aims to provide member cities with a platform to exchange information on the sustainable management
of urban rivers.
x Initially, the Alliance was launched for Ganga basin cities across India only, but it has been extended to
include other cities of India (not other countries) beyond the basin. Hence statement 3 is not correct.
x The participating cities include Dehradun, Haridwar, Patna, Bhubaneswar, Chennai, Hyderabad,
Pune, Udaipur, and Vijayawada, among others. The National Mission for Clean Ganga (NMCG) and
National Institute for Urban Affairs (NIUA) have collaborated to launch the RCA.
x The primary objectives of the RCA are to adopt national policies and instruments with key river-related
directions, develop urban river management plans and city-specific sectoral strategies for sustainable
management of urban rivers, and provide a platform for member cities to exchange information.
Hence statement 1 is correct.

 23 www.visionias.in ©Vision IAS


.
 Q 56.A
x The caste system was the basis of social organisation under the Cholas. Society was divided into a number
of social groups or castes. Each caste was hereditary and constituted an occupational group.
 x Brahmins and Kshatriyas enjoyed special privileges. The inscriptions of the later period of Chola rule
mention two major divisions among the castes – Valangai and Idangai castes. However, there was
cooperation among various castes and sub-castes in social and religious life. Hence statement 1 is correct.
x The position of women did not improve under the Cholas. The practice of ‘sati’ was prevalent among the
royal families. Some of the royal ladies who committed sati include Sundara Chola queen Vanavan
Mahadevi, the mother of Rajaraja Chola I (10th century), and Viramahadevi, the queen of Rajendra Chola
I (11th century), both of whom ascended the pyre upon their husband's deaths.
x A few of the queens of the chieftains also committed sati; for example, Gangamadevi, queen of Veerachola
Ilangovelar, the Kodumbalur chieftain, as evident from the Alur inscription. Hence statement 2 is not
correct.

Q 57.C
x Recent context: A sunspot, called AR3038, grew to almost twice its size in the span of 24 hours.
x Sunspots are areas that appear dark on the surface of the Sun. They appear dark because they are
cooler than other parts of the Sun’s surface. Hence statement 1 is not correct.
x Sunspots are relatively cool as they form at areas where magnetic fields are particularly strong thus
keeping some of the heat within the Sun from reaching the surface. Hence statement 2 is not correct.
x Magnetic field lines near sunspots can cause a sudden explosion of energy called a solar flare. Hence
statement 3 is correct.

Q 58.A
x An overwhelming majority (99 percent) of animals and Nearly all plants cannot maintain a constant internal
environment.
x Their body temperature changes with the ambient temperature. In aquatic animals, the osmotic
concentration of the body fluids changes with that of the ambient water osmotic concentration. These
animals and plants are simply conformers. Considering the benefit of a constant internal environment to the
organism, we must ask why these conformers had not evolved to become regulators.
x Thermoregulation is energetically expensive for many organisms. This is particularly true for small animals
like shrews and hummingbirds. Heat loss or heat gain is a function of surface area. Since small animals
have a larger surface area relative to their volume, they tend to lose body heat very fast when it is
cold outside; then they have to expend much energy to generate body heat through metabolism. This
is the main reason why very small animals are rarely found in polar regions. During the course of
evolution, the costs and benefits of maintaining a constant internal environment are taken into consideration.
Some species have evolved the ability to regulate, but only over a limited range of environmental conditions,
beyond which they simply conform. If the stressful external conditions are localized or remain only for a
short duration, the organism has two other alternatives. Hence, option (a) is the correct answer.

Q 59.C
x The words ‘council of ministers’ and ‘cabinet’ are often used interchangeably though there is a definite
distinction between them. They differ from each other in respects of composition, functions, and role. These
differences are:

 24 www.visionias.in ©Vision IAS


.


x Hence option (c) is the correct answer.

Q 60.A
x Recently an international team of scientists made a breakthrough in retaining the quantum coherence
of quantum dot spin qubits.
x Quantum coherence: Ability of a quantum state to maintain its entanglement and superposition in the face
of interactions.
o It deals with the idea that all objects have wave-like properties. Hence option (a) is the correct
answer.
o If an object’s wave-like nature split in two, then two waves may coherently interfere with each other in
such a way to form a single state that is a superposition of 2 states (a qubit is a superposition of the 0
and 1 states).

Q 61.B
x The North Anatolian Fault is a major active fault system that runs east-west across northern Turkey,
extending for about 1,200 kilometers from the eastern end of the Sea of Marmara to the eastern part of
Turkey. It is a transform fault that separates the Anatolian Plate from the Eurasian Plate, in northern
Turkey and it is a part of the larger complex of fault systems that define the tectonic boundary between the
African, Eurasian, and Arabian plates. Hence statement 1 is not correct and 2 is correct

 25 www.visionias.in ©Vision IAS


.
 x The North Anatolian Fault is a major source of seismic activity in Turkey, and it has produced many
large earthquakes over the past century, including the devastating earthquakes of 1939, 1942, 1944, 1947,
1957, 1967, 1999, and 2020. These earthquakes have caused significant damage and loss of life in the
 region.
x The fault system consists of a series of interconnected faults that exhibit complex behavior, with
segments that can rupture independently or together in large earthquakes. The fault system also
exhibits a pattern of eastward propagation of seismic activity, with earthquakes moving progressively
eastward along the fault over time.
x The North Anatolian Fault is closely monitored by the Turkish Disaster and Emergency Management
Authority (AFAD) and other agencies to assess the risk of seismic activity and provide early warning of
potential earthquakes. Studies of the fault system also provide important insights into the behavior of
complex fault systems and the nature of seismic activity in general.

Q 62.C
x Union Cabinet recently gave the 14th extension to the Justice Rohini Commission, set up to examine
issues related to sub-categorization of OBCs, to submit its report by July 31, 2023. Hence statement 1
is correct.
x Commission was formed in 2017 under Article 340.
o Article 340 empowers President to appoint a commission to investigate the conditions of
backward classes. Hence statement 3 is correct.
x In 2015, NCBC had proposed that OBCs be divided into following three categories:
o Extremely Backward Classes (EBCGroup A) facing social, educational and economic backwardness
even within the OBCs, consisting of aboriginal tribes, nomadic and seminomadic tribes who have been
carrying on with their traditional occupations.
o More Backward Classes (MBCGroup B) consisting of vocational groups carrying on with their
traditional occupations.
o Backward Classes (BC-Group C) comprising of those comparatively more forward.
x According to NCBC, 11 states/UTs (Andhra Pradesh, Telangana, Puducherry, Karnataka, Haryana,
etc.) have subcategorized OBC for reservations in state-government owned institutions. Hence
statement 2 is not correct.
x Mandate of Justice Rohini Commission
o Examine extent of inequitable distribution of reservation benefits (i.e. 27% reservation in jobs and
education) among castes or communities with reference to central OBC list.
o Work out the mechanism, criteria, norms and parameters in a scientific approach for sub-categorization
of OBCs.

 26 www.visionias.in ©Vision IAS


.


Q 63.B
x The recent initiative of the launch of Open Network for Digital Commerce (ONDC) is playing a
significant role in democratizing digital payments, enabling interoperability, and bringing down transaction
costs.
x ONDC provides better market access to sellers and helps bring the country's remotest corners into
the e-Commerce framework by empowering them with digitisation. Hence, option (b) is the correct
answer.
x ONDC aims to go beyond the current platform-centric digital commerce model where the buyer and seller
can use the same platform or application for transactions. ONDC is a network-based open protocol that
would connect all the buyers and sellers on the network thus bringing better visibility across the Business-
to-Consumer (B2C) & Business-to-Business (B2B) landscape. In the ONDC framework, consumers can
potentially discover any seller, product, or service by using any compatible application or platform.
Q 64.A
x Pandit Mahesh Chandra Bhattacharyya (22 February 1836 – 12 April 1906) was an Indian scholar
of Sanskrit, and the principal of the Sanskrit College between 1876 and 1895.
x A friend and colleague of Ishwar Chandra Vidyasagar, he played an important role in the Bengal
Renaissance. He was one of the most eminent Bengalis in Kolkata of the nineteenth century.
x In 1876, he took over as Sanskrit College's principal after Prasanna Kumar Sarbadhikary.
x During his 19 years as principal, he took the initiative to start the Sanskrit Title Examination, which
awards titles to deserving students in specialised Sanskrit departments.
x In his hometown of Narit, he founded a secondary Anglo-Sanskrit school that is still in operation today as
Narit Nyayratna Institution.
x He revised Kavya Prakas, Mimansa Darshan, and the Black Yajur Veda with extensive comments.
Comments on Dayananda Saravati's Veda-Bhashya, Tulasidharan Mimansa, The Authorship of
Mrichchhakatika, and Lupta Samvatsara are only a few of the pamphlets he produced.
x The renowned Vedic scholar Swami Dayanand responded to the questions posed by Mahesh Chandra
Nyayaratna in 1880 by writing a book under the title Bhranti Nivarana.
x He made significant contributions to the general growth of Sanskrit study as well as the relief of hunger,
the promotion of education, and the opening up of communication channels through financial and other
methods.
x He played a key role in creating roads and infrastructure, including tramways, in his hometown of Howrah
in addition to significantly enhancing the roads and infrastructure in and around Narit.
x On February 16, 1887, in honour of Queen Victoria's Jubilee, the title of Mahamahopadhyay was
given to him as a personal accolade for excellence in oriental learning. It gave him the rank in Durbar
right after titular Rajas.
x Hence, option (a) is the correct answer.

Q 65.A
x A Depository is an institution that holds financial securities like shares, debentures, and mutual funds
in electronic form (De-Materialized form). It also facilitates the trade of these securities on various stock
exchanges. It is responsible for the maintenance of ownership records and facilitation of trading in
dematerialized securities. Hence statement 1 is correct.

 27 www.visionias.in ©Vision IAS


.
 x A depository works as a link between the listed companies which issue shares and shareholders. It issues
these shares through agents associated with it called depository participants or DPs. A DP can be a bank,
financial institution, broker, or any entity eligible as per SEBI norms and is responsible for the final transfer
 of shares from the depository to investors. The investor, at the end of a transaction, receives a confirmation
from the depository.
x At present two Depositories viz. National Securities Depository Limited (NSDL) and Central
Depository Services (India) Limited (CDSL) are registered with SEBI. Depository Act of 1996 paved
the way for the establishment of two depositories in India, namely National Securities Depository Limited
(NSDL) which is promoted by the National Stock Exchange, Industrial Development Bank of India, and
Unit Trust of India among others. The other depository is the Central Depository Services Limited (CDSL)
which is promoted by the Bombay Stock Exchange, State Bank of India, and Bank of India among
others. Hence statement 2 is correct.
x The Securities and Exchange Board of India is responsible for the registration, regulation, and
inspection of the depository. A depository participant is also answerable to the SEBI. It can be operational
only after registration with SEBI post recommendation by NSDL or CDSL. Hence statement 3 is not
correct.

Q 66.D
x Recent context: Dr Dilip Mahalanabis who pioneered the proper practical, emergency use of oral
rehydration solution, commonly known by its abbreviation “ORS,” for diarrhoeal diseases passed
away recently.
x Oral rehydration salts (ORS):
o ORS are a mixture of electrolytes (salts) and carbohydrates (in the form of sugar) dissolved in
water. Hence statement 1 is correct.
o They are used to replace salts and water that the body loses during dehydration caused by gastroenteritis,
diarrhoea or vomiting. Hence statement 3 is correct.
o The electrolytes are potassium and sodium. These components maximize fluid absorption in the
gastrointestinal tract. Hence statement 2 is correct.
o The gastrointestinal tract relies on sodium-glucose cotransporters (SGLTs), which are carrier proteins
in the intestinal cells.
o Cotransporters help move substances across membranes.
o Specifically, SGLTs pair together sodium and glucose transport in the small intestine. This allows
glucose to increase the absorption of fluids.
o Additionally, sodium needs glucose to be properly absorbed. Therefore, ORS contains both glucose and
sodium.

Q 67.D
x Articles 239 to 241 in Part VIII of the Constitution deal with the union territories.
x Even though all the union territories belong to one category, there is no uniformity in their administrative
system.
x The President can specify the designation of an administrator; it may be Lieutenant Governor or Chief
Commissioner or Administrator.
x At present, it is Lieutenant Governor in the case of Delhi, Puducherry, Andaman and Nicobar
Islands, Jammu and Kashmir and Ladakh and Administrator in the case of Chandigarh, Dadra and
Nagar Haveli, Daman and Diu and Lakshadweep. Hence, statement 1 is not correct.
x The Parliament can make laws on any subject of the three lists (including the State List) for the union
territories. This power of Parliament also extends to Puducherry, Delhi and Jammu and Kashmir, which
have their own local legislatures. This means that the legislative power of Parliament for the union
territories on subjects of the State List remains unaffected even after establishing a local legislature for
them. Hence, statement 2 is not correct.
x The Constitution does not contain any separate provisions for the administration of acquired
territories. But, the constitutional provisions for the administration of union territories also apply to the
acquired territories. Hence, statement 3 is correct.

Q 68.C
x Kai Chutney of Mayurbhanj district, Odisha is aiming to make a big leap from the remote tribal villages
to the global food tables as the dish has applied for a Geographical Identification (GI) registration.
Hence option (c) is the correct answer.

 28 www.visionias.in ©Vision IAS


.
 x Kai Chutney:
o The Kai Chutney is prepared from the Weaver ants and is popular in Odisha’s Mayurbhanj district
among the people, mostly tribals like Bathudi tribe.
 o It has been a prominent part of cuisine for hundred of years for the tribes of Similipal biosphere
reserve spread across Odisha's Mayurbanj, Keonjhar and Sundagarh districts.
o When required, leafy nests of ants are plucked from their host trees and collected in a bucket of water
before sorting and separating from leaves and debris.
x Red Weaver ants:
o Kai (Red Weaver Ant) Ants, scientifically called Oecophylla smaragdina, are abundantly found in
Mayurbhanj throughout the year. They construct nests with the leaves of host trees. The nests are
strong enough to withstand wind and are impermeable to water.
o Kai’s nests are usually elliptical in shape and range in size from a single small leaf folded and bound
onto itself to large nests consisting of many leaves and which measure over half a meter in length.
o They feed on small insects and other invertebrates, their prey being mainly beetles, flies, and
hymenopterans. Kais are bio-control agents. They are aggressive and prey on most arthropods entering
their territory.

Q 69.B
x The Global Biofuel Alliance is one of the priorities under India’s G20 Presidency and was announced by
India’s Petroleum and Natural Gas Minister Hardeep Sing Puri during India Energy Week 2023. Hence
statement 1 is not correct.
o India is leveraging its presidency of the G20 to form a global biofuels alliance on the lines of the
International Solar Alliance (ISA).
o India, Brazil and the US, as leading biofuel producers and consumers in the world, will work together
during the next few months towards the development of a Global Biofuels Alliance along with other
interested countries.
x This alliance will be aimed at facilitating cooperation and intensifying the use of sustainable biofuels,
including in the transportation sector. Hence statement 2 is correct.
o The focus of the alliance will be on strengthening markets, facilitating global biofuels
trade, development of concrete policy lesson-sharing and provision of technical support for national
biofuels programs worldwide.
o The alliance shall work in collaboration with and complement the relevant existing regional and
international agencies as well as initiatives in the bioenergy, bioeconomy, and energy transition fields
more broadly, including the Clean Energy Ministerial Biofuture Platform, the Mission Innovation
Bioenergy initiatives, and the Global Bioenergy Partnership (GBEP).

Q 70.C
x Recently Energy Conservation (Amendment) Act, 2022 came into force.
x Its key features:
o It empowers the central government to specify a carbon credit trading scheme. Hence statement
1 is correct.
o Carbon credit implies a tradable permit to produce a specified amount of CO2 or other GHGs. Central
government or any authorized agency may issue carbon credit certificates to entities registered and
compliant with the scheme.
o Importantly, energy savings or carbon credits can now be bought on a voluntary basis, opening
additional financing for clean technologies.
o The export of excess carbon credits will be allowed, but the Government’s primary focus will be
on using the carbon market to meet the country’s NDCs. Hence, statement 2 is not correct.
x A government is empowered to set requirements for designated consumers to meet a minimum share of
energy consumption from non-fossil sources like green hydrogen, green ammonia, etc. Failure to meet
obligation will be punishable with a penalty of up to Rs 10 lakh.
x ‘Energy Conservation and Sustainable Building Code’ to replace Energy Conservation Code for
buildings. This new code will provide norms for energy efficiency and conservation, the use of renewable
energy, and other requirements for green buildings.
o Also applicable to the office and residential buildings meeting the above criteria. It also empowers state
governments to lower the load thresholds.
x Expands the scope of energy consumption standards to include vehicles (as defined under the Motor
Vehicles Act, 1988), and vessels (including ships and boats), in addition to equipment and appliances.
Hence statement 3 is correct.
 29 www.visionias.in ©Vision IAS
.
 Q 71.D
x The United Nations Ocean Conference was a high-level conference aimed to support the conservation
and sustainable use of the oceans, seas, and marine resources.
 x The UN Ocean Conference, co-hosted by the Governments of Kenya and Portugal, came at a critical
time as the world is seeking to address the many of the deep-rooted problems of our societies. Hence
statement 1 is not correct.
x Lisbon declaration: It sets out non-binding targets on 198 members of the United Nations to conserve
oceans. They also recognised that developing countries, particularly small island developing states and least
developed countries, need assistance with capacity building. Hence statement 2 is not correct.
x The 2022 UN Ocean Conference is in line with the Sustainable Development Goal (SDG) 14. Sustainable
Development Goal 14: Life Below Water: the need to conserve and sustainably use the world’s oceans,
seas and marine resources.

Q 72.B
x NCRB reported about 500% rise in cases under hate speech law in seven years. Hate speech has not been
defined in any law in India.
x However, hate speech, generally incitement to hatred primarily against a group of persons, dealt with IPC
section like 153A, 295A etc.

x Hence option (b) is the correct answer.


x Additional Information
x Section 8 in The Representation of the People Act, 1951 provides for disqualification on conviction for
certain offences
x Under Section 8 (1), a person convicted of an offence punishable under
o offence of promoting enmity between different groups on ground of religion, race, place of birth,
residence, language, etc., and doing acts prejudicial to the maintenance of harmony
o offence of bribery
o offence of undue influence or personation at an election
o offences relating to rape
o offence of cruelty towards a woman by husband or relative of a husband
o offence of making statement creating or promoting enmity, hatred or ill-will between classes or offence
relating to such statement in any place of worship or in any assembly engaged in the performance of
religious worship or religious ceremonies
o offence of importing or exporting prohibited goods) of the Customs Act, 1962
o Protection of Civil Rights Act, 1955 (22 of 1955), which provides for punishment for the preaching and
practice of “untouchability”, and for the enforcement of any disability arising therefrom
o Foreign Exchange (Regulation) Act, 1973
o Narcotic Drugs and Psychotropic Substances Act, 1985
o offence of contravention of the provisions of sections 3 to 6 of the Religious Institutions (Prevention of
Misuse) Act, 1988
o offence of removal of ballot papers from polling stations
o offence of booth capturing
 30 www.visionias.in ©Vision IAS
.
 o offence of fraudulently defacing or fraudulently destroying any nomination paper of this Act
o offence of conversion of a place of worship of the Places of Worship (Special Provisions) Act, 1991]
o offence of insulting the Indian National Flag or the Constitution of India
 o offence of preventing singing of National Anthem of the Prevention of Insults to National Honour Act,
1971
o Prevention of Corruption Act, 1988
o Prevention of Terrorism Act, 2002
x Under Section 8(2), A person convicted for the contravention of
o any law providing for the prevention of hoarding or profiteering
o any law relating to the adulteration of food or drugs
o any provisions of the Dowry Prohibition Act and sentenced to imprisonment for not less than six
months, shall be disqualified from the date of such conviction and shall continue to be disqualified for
a further period of six years since his release.
x Under section 8(3)
o A person convicted of any offence and sentenced to imprisonment for not less than two years [other
than any offence referred to in sub-section (1) or sub-section (2) given above] shall be disqualified from
the date of such conviction and shall continue to be disqualified for a further period of six years since
his release.
x Section 8A provides for disqualification in The Representation of the People Act, 1951 on ground of corrupt
practices
o (1) The case of every person found guilty of a corrupt practice by an order under section 99 shall be
submitted, as soon as may be after such order takes effect, by such authority as the Central Government
may specify in this behalf, to the President for determination of the question as to whether such person
shall be disqualified and if so, for what period
o (2) Any person who stands disqualified under section 8A of this Act as it stood immediately before the
commencement of the Election Laws (Amendment) Act, 1975 (40 of 1975), may, if the period of such
disqualification has not expired, submit a petition to the President for the removal of such
disqualification for the unexpired portion of the said period.
o (3) Before giving his decision on any question mentioned in sub-section (1) or on any petition submitted
under sub-section (2) given above, the President shall obtain the opinion of the Election Commission
on such question or petition and shall act according to such opinion.

Q 73.C
x Recent Context: The Ministry of Electronics and IT (MeitY) has declared the IT resources of ICICI
Bank, HDFC Bank and UPI managing entity NPCI as 'critical information infrastructure', implying
any harm to them can have an impact on national security.
x Critical information infrastructure (CII) refers to the underlying technology and systems that are
essential to the functioning of a society and economy. These systems provide crucial services, such as
telecommunications, energy, transportation, finance, healthcare, and government services. The failure or
disruption of these systems can have severe consequences, including economic losses, threats to public
safety.
x The Information Technology (IT) Act, enacted in 2000, defines Critical Information Infrastructure
(CII) in Section 70(1) as "such computer resource as is vital to the functioning of the country and the
damage to, or the unauthorized access to, which shall have a debilitating impact on national security,
economy, public health, or safety. Hence statement 1 is correct.
x Under the IT Act of 2000, the government has the power to declare any data, database, computer
resource, or IT network as Critical Information Infrastructure (CII) if it is deemed vital to the
functioning of the country. Hence statement 2 is correct.

Q 74.D
x The Ministry of Corporate Affairs (MCA) has published a draft framework for cross-border insolvency
proceedings based on the UNCITRAL (United Nations Commission on International Trade Law) model
under the Insolvency and Bankruptcy Code.
x Cross-border insolvency proceedings are relevant for the resolution of distressed companies with
assets and liabilities across multiple jurisdictions. A framework for cross-border insolvency proceedings
allow for the location of such a company’s foreign assets, the identification of creditors and their claims,
and establishing of payment towards claims as well as a process for coordination between courts in different
countries.

 31 www.visionias.in ©Vision IAS


.
 x While foreign creditors can make claims against a domestic company, the IBC currently does not allow for
automatic recognition of any insolvency proceedings in other countries. In the case of Jet Airways, when
one of the company’s aircraft was grounded in Amsterdam over non-payment of dues to a European cargo
 firm, the National Company Law Tribunal had declined to “take on record” any orders of a foreign court
regarding domestic insolvency proceedings in the absence of enabling provision in the IBC.
x The UNCITRAL model is the most widely accepted legal framework to deal with cross-border
insolvency issues. It has been adopted by 49 countries, including the UK, the US, South Africa, South
Korea, and Singapore.
x The law allows automatic recognition of foreign proceedings and rulings given by courts in cases where the
foreign jurisdiction is adjudged as the Centre Of Main Interests (COMI) for the distressed company.
Recognition of foreign proceedings and reliefs is left to the discretion of domestic courts when foreign
proceedings are non-main proceedings. The COMI for a company is determined based on where the
company conducts its business on a regular basis and the location of its registered office.
x Hence option (d) is the correct answer.

Q 75.A
x India will be one of the main drivers of global insurance industry growth over the next decade. Indian
Insurance Market is the 10th largest in the world and is poised to become the 6th largest by 2032, ahead of
Germany, Canada, Italy, and South Korea.
x Insurance penetration in India increased steadily from 2.7 per cent around the turn of the millennium to 4.2
per cent in 2020 and remained the same in 2021. Life insurance penetration in India was 3.2 per cent in
2021, almost twice more than the emerging markets and slightly above the global average. However, most
life insurance products sold in India are savings-linked, with just a small protection
component. Hence, households remain exposed to a significant financing gap in the event of the premature
death of the primary breadwinner.
x A rapidly maturing insurance market has provided a lucrative opportunity to the government to
privatize its stake in the insurance business. Accordingly, the General Insurance Business
(Nationalisation) Amendment Act, 2021 allows the central government to pare its stake to less than
51 per cent of the equity capital in a specified insurer. Hence statement 1 is correct.
x In line with the Government of India’s vision towards Financial Inclusion and a strong emphasis on
accelerating reforms, IRDAI, during FY23, has implemented the following measures to increase
accessibility, innovation, competition, distribution efficiency, and choice availability while mainstreaming
technology and moving towards a principle-based regime: -
o Easy entry into the insurance sector: A Single Window NOC Portal (www.noc.irdai.gov.in) was
launched to facilitate the incorporation of an insurer by making the NOC available in a hassle-free and
timely manner.
o Quick launch of Insurance Products: Insurers can now launch all Health & General Insurance
products, as well as the majority of Life Insurance products, without seeking prior approval from
IRDAI, thereby reducing the time taken to launch a new product in the market from a few months
to a few days.
o Ease of doing business: IRDAI has taken several actions to lessen the burden of compliance. In this
direction, so far, 70 returns have been rationalized, and about 85 circulars have been repealed while
dispensing with the prior approval requirements in certain identified areas.
o Providing further impetus to the industry: Given that the sector has reached a level of maturity that may
not necessitate closer supervision, more flexibility to the regulated entities in the areas of operational
and business decisions is being considered.
x The key tax change proposed in the Finance Bill 2023 with implications to the insurance business are:
o Tax exemption removed for life insurance policies with high-value insurance premium: Mooted
as part of an emphasis on better targeting of tax concessions and exemptions, the proposal means that
only income from life insurance policies with an aggregate premium up to ₹5 lakh will be exempt from
taxation. Hence statement 2 is not correct.
o FATCA / CRS reporting: In order to penalize false self-certification provided by policyholders which
in turn leads to furnishing of incorrect statements under section 285BA of the Act by reporting life
insurance companies (i.e. reporting in Form 61B), it is proposed to amend section 271FAA of the Act.
It is proposed to prescribe a penalty of Rs.5,000 on the reporting life insurance companies if there is
any inaccuracy in the statement of financial transactions submitted by a prescribed reporting financial
institution and such inaccuracy is due to false or inaccurate information submitted by the policy holder.

 32 www.visionias.in ©Vision IAS


.
 Q 76.D
x Buddhist Councils
o The first Buddhist Council was held at Rajagraha under the chairmanship of Mahakasapa
 immediately after the death of Buddha. Its purpose was to maintain the purity of the teachings of
the Buddha. Hence pair 1 is correctly matched.
o At this council, Ananda composed the Suttapitaka (Buddha’s Teachings) and Mahakassapa composed
the Vinaypitaka (monastic code).
o The second Buddhist Council was convened at Vaisali around 383 B.C. The main objective was to
discuss ten disputed points under the Vinaypitaka. Hence pair 2 is correctly matched.
o The first major split happened here – two groups that would later evolve into Theravada and
Mahayana. The first group was called Thera (meaning Elder in Pali). They wanted to preserve the
teachings of Buddha in the original spirit. The other group called Mahasanghika (Great Community)
interpreted the Buddha’s teachings more liberally.
o The third Buddhist Council was held at Pataliputra under the patronage of Asoka. Moggaliputta Tissa
presided over it. The final version of Tripitakas (Abhidhamma Pitaka) was completed in this
council. Hence pair 3 is correctly matched.
o The fourth Buddhist Council was convened in Kashmir by Kanishka under the chairmanship of
Vasumitra. Asvagosha participated in this council. The new school of Buddhism called Mahayana
Buddhism came into existence during this council. Hence pair 4 is correctly matched.
o The Buddhism preached by the Buddha and propagated by Asoka was known as Hinayana.
o The Buddhist texts were collected and compiled some five hundred years after the death of the Buddha.
They are known as the Tripitakas, namely the Sutta, the Vinaya and the Abhidhamma Pitakas. They are
written in the Pali language.

Q 77.B
x Towards the close of the nineteenth century, Indian painting, as an extension of the Indian miniature
painting, snapped and fell on the decline and degenerated into feeble and unfelt imitation largely due to
historical reasons, both political and sociological, resulting in the creation of a lacuna which was not filled
until the early years of the twentieth century, and even then not truly.
o There was only some minor artistic expression in the intervening period by way of the ‘Bazar’
and ‘Company’ styles of painting, apart from the more substantial folk forms which were alive
in many parts of the country.
o Then followed the newly ushered Western concept of naturalism, the foremost exponent of which
was Raja Ravi Varma. This was without parallel in the entire annals of Indian Art notwithstanding
some occasional references in Indian literature of the idea of ‘likeness’. Hence statement 1 is not
correct.
o The work 'Lady in the Moonlight' is a famous painting of Raja Ravi Varma. In this painting, a lady
sits alone in the moonlight. It's a classic painting with extraordinary light effects.
x An attempt to stem this cultural morass was made by Abanindranath Tagore under whose inspired
leadership came into being a new school of painting which was distinctly nostalgic and romantic to start
with. It held its way for well over three decades as the Bengal School of Painting, also called the
Renaissance School or the Revivalist School – it was both.
o Bharat Mata is a work painted by the Indian painter Abanindranath Tagore in 1905. The work
depicts Bharat Mata, or Mother India, in the style of a Hindu Goddess.
o The painting was the first illustrated depiction of the concept and was painted with Swadesh ideals
during the larger Indian Independence movement. Bharat Mata, depicts a saffron-clad woman, dressed
like a sadhvi, holding a book, sheaves of paddy, a piece of white cloth and a garland in her four hands.
The painting holds historical significance as it is one of the earliest visualizations of Bharat Mata, or
"Mother India
x Jamini Roy was a contemporary of the artists like Nandlal Bose, but he, more than anyone else, sought
an entirely different path of expression, which had a tremendous impact on subsequent painters, deeply
inspired by the Bengal folk tradition. His images and ideas as in this painting of 'Pujarinis' are direct,
singularly stylised, and conceived in emphatic flat spaces and strong lines. Hence statement 3 is
correct.
x Amrita Sher-Gil was one of the most famous painters of India, often referred to as India's Frida Kahlo for
aesthetically blending traditional and Western art forms.
o She was influenced by the Mughal as well as the Ajanta paintings.
o In 1937, she began her journey to the southern parts India and was deeply moved by the plight of many
villagers and unprivileged people. This started reflecting in her works and eventually gave rise to
 33 www.visionias.in ©Vision IAS
.
 paintings such as ‘Brahmacharis’, ‘Bride's Toilet’ and ‘South Indian Villagers Going to Market’.
Hence statement 2 is correct.

 Q 78.B
x Tropics account for greater biological diversity.
x Various hypotheses have been given. Some important ones are:
o Speciation is generally a function of time, unlike temperate regions subjected to frequent glaciations in
the past, tropical latitudes have remained relatively undisturbed for millions of years and thus, had
a long evolutionary time for species diversification. Hence, option 1 is correct.
o Tropical environments, unlike temperate ones, are less seasonal, relatively more constant, and
predictable. Such constant environments promote niche specialization and lead to greater species
diversity. Hence, option 2 is not correct.
o There is more solar energy available in the tropics, which contributes to higher productivity; this in
turn might contribute indirectly to greater diversity. Hence, option 3 is correct.

Q 79.D
x In Anoop Baranwal vs Union of India – a judgment delivered on March 2, 2023 – a Constitution Bench
of the Supreme Court unanimously held that the selection of the Chief Election Commissioner and the
Election Commissioners would be done by a three-member Committee consisting of the Prime
Minister, the Leader of the Opposition (or the leader of the largest opposition party in Parliament),
and the Chief Justice of India. The Court, thus, altered the present mode of selection, where the CEC is
appointed by the President, acting on the advice of the Prime Minister.
x Hence option (d) is the correct answer.

Q 80.B
x Nyaya is one of the six classical schools of Indian philosophy. It is primarily analytic and therefore more
concerned with logic and epistemology than ethics. The Nyaya school of philosophy was founded by the
sage Gotama, also known as Gautama or Aksapada. He is believed to have lived in the 3rd century BCE.
Gautama's work on Nyaya philosophy is presented in a text called the Nyaya Sutras. Hence statement 1 is
correct.
x The ultimate aim of Nyaya philosophy is to alleviate human suffering by gaining knowledge of reality.
Liberation is achieved through valid knowledge, which is referred to as prama, and invalid knowledge is
referred to as aprama. Nyaya emphasizes the importance of attaining valid knowledge, as it is believed to
lead to the true apprehension of an object and a clear understanding of reality.
x The Nyaya school recognizes four valid means of knowledge: perception (pratyaksha), inference
(anumana), comparison (upamana), and testimony (shabda).
o Perception is the direct apprehension of an object through the senses. Inference is the process of arriving
at a conclusion based on premises that are known or observed. Comparison is the process of determining
similarities and differences between two or more objects. Testimony is the knowledge gained from the
words of trustworthy persons.
o These four means of knowledge are considered reliable and valid for obtaining true knowledge. Invalid
knowledge involves memory, doubt, error, and hypothetical argument. Hence statement 2 is not
correct.
x The theory of causation is an important component of Nyaya philosophy. According to Nyaya, causation
involves two important factors – cause and effect. A cause is defined as an unconditional and invariable
antecedent of an effect, while an effect is defined as an unconditional and invariable consequent of a cause.
x The Nyaya theory of causation is known as ‘astkaryavada’ or ‘armbhavada’, which suggests that an effect
is produced by a cause, but the effect and the cause are not the same. Nyaya philosophers argue that every
effect is a new product that did not exist previously in the cause.
o For example, a pot is made of clay. Here ‘clay’ is the cause and ‘pot’ is its effect. According to Nyaya,
pot is a fresh creation, a new beginning that did not exist before in the clay. In this way, Nyaya uphold
the theory of ‘Asatkaryavada’. Hence statement 3 is correct.

Q 81.B
x According to the Ministry of Home Affairs, National Automated Fingerprint Identification System
(NAFIS), which was developed by the National Crime Records Bureau (NCRB), would help in the quick
and easy disposal of cases with the help of a centralized fingerprint database in April this year, Madhya
Pradesh became the first state in the country to identify a deceased person through NAFIS.

 34 www.visionias.in ©Vision IAS


.
 x NAFIS is a country-wide searchable database of crime- and criminal-related fingerprints. It is
conceptualized and managed by the NCRB at the Central Fingerprint Bureau (CFPB). Hence
statement 1 is correct and statement 3 is not correct.
 x The web-based application functions as a central information repository by consolidating fingerprint data
from all states and Union Territories.
x According to a 2020 report by the NCRB, it enables law enforcement agencies to upload, trace, and retrieve
data from the database in real-time on a 24×7 basis.
x NAFIS assigns a unique 10-digit National Fingerprint Number (NFN) to each person arrested for a
crime. Hence statement 2 is correct.
o This unique ID will be used for the person’s lifetime, and different crimes registered under different
FIRs will be linked to the same NFN.
x By automating the collection, storage, and matching of fingerprints, along with digitizing the records of
fingerprint data, NAFIS will “provide the much-needed unique identifier for every arrested person in
the CCTNS (Crime and Criminal Tracking Network & Systems) database as both are connected at
the backend.

Q 82.A
x If the stressful external conditions are localised or remain only for a short duration, the organism has two
other alternatives.
x Suspend: In bacteria, fungi and lower plants, various kinds of thick-walled spores are formed which help
them to survive unfavourable conditions – these germinate on the availability of a suitable environment.
x In higher plants, seeds and some other vegetative reproductive structures serve as means to tide over periods
of stress besides helping in dispersal – they germinate to form new plants under favourable moisture and
temperature conditions. They do so by reducing their metabolic activity and going into a date of
‘dormancy’. In animals, the organism, if unable to migrate, might avoid the stress by escaping in time.
x The familiar case of Polar Bears going into hibernation during winter is an example of escape in
time. Hence, pair 1 is correctly matched.
o Hibernation is when an animal slows its heart rate to save energy and survive the winter without
eating much. Some animals just slow down and move less frequently during hibernation, but
others go into a deep sleep and don't wake up till spring.
x Some snails and fish go into aestivation to avoid summer–related problems-heat and desiccation. Hence,
pair 2 is not correctly matched.
o Aestivation is a state of animal dormancy, similar to hibernation, although taking place in the
summer rather than the winter. Aestivation is characterized by inactivity and a lowered metabolic
rate, that is entered in response to high temperatures and arid conditions. It takes place during times of
heat and dryness, the hot dry season, which are often the summer months.
x Under unfavourable conditions, many zooplankton species in lakes and ponds are known to
enter diapause, a stage of suspended development. Hence, pair 3 is not correctly matched.
o Diapause is a phase of dormancy in insects and zooplankton. Insects can undergo diapause during
the summer or winter as eggs, larvae, pupae, or adults and may be used as a predator avoidance
mechanism.

Q 83.C
x Pennar river basin is located in peninsular India, and it extends over the states of Andhra Pradesh and
Karnataka having an area of 55,213 Sq.km with maximum length and width of 433 km and 266 km. Hence
option (c) is the correct answer.
x The fan-shaped basin is bounded by the Erramala range on the north, by the Nallamala and Velikonda
ranges of the Eastern Ghats on the east, by the Nandidurg hills on the south, and by the narrow ridge
separating it from the Vedavati valley of the Krishna Basin on the west. The other hill ranges in the basin
to the south of the river are the Seshachalam and Paliconda ranges.
x The Pennar (also known as Uttara Pinakini) is one of the major rivers of the peninsula. The Pennar rises
in the Chenna Kasava hill of the Nandidurg range, in the Chikkaballapura district of Karnataka and flows
towards the east, and eventually drains into the Bay of Bengal in the Nellore district of Andhra Pradesh.
x The principal tributaries of the river joining from the left are the Jayamangali, the Kunderu, and the
Sagileru whereas the Chiravati, the Papagni, and the Cheyyeru join it from the right.

 35 www.visionias.in ©Vision IAS


.


Q 84.A
x The Union Budget 2023 has doubled the PM Gati Shakti National Master Plan to States from ₹5,000 crores
to ₹10,000 crores and has announced an outlay of ₹2.4 lakh crore for the Indian Railways. The plan is a
“transformative approach for economic growth and sustainable development depends on the engines of
roads, railways, airports, ports, mass transport, waterways and logistics infrastructure”. The Railways offer
an efficient and economic mode of logistics movement given their pan-India network and can play an
important role in enabling a coordinated and integrated logistics system.
x The National Rail Plan envisages that the share of freight traffic by rail should go up from the current
share of 27% to 45% by 2030. The construction of Dedicated Freight Corridors (DFCs) on the important
high-density route is an important policy measure by Indian Railways to arrest the trend of falling market
share of railways in the country and also will shift the advantage in favour of rail transport. Hence
statement 3 is not correct.
x Currently, the modal mix in terms of freight movement (by volume) is skewed by a considerable
extent towards road transport, with 65% of freight movement by road. The effect is an increased
burden on roads, and, therefore, significant congestion, increased pollution, and resultant logistics cost
escalations. The increased adoption of the railways as a mode for cargo movement is crucial to improve
India’s logistics competitiveness. Hence statement 1 is correct.

x In 2020-21, coal constituted 44% of the total freight movement of 1.2 billion tonnes, followed by iron ore
(13%), cement (10%), food grains (5%), fertilizers (4%), iron and steel (4%), etc. Transportation of non-
bulk commodities accounts for a very small share in the rail freight movement. Hence statement 2 is
correct.
x The convenience of moving non-bulk commodities in containers has led to an increase in containerized
traffic over the last decade, growing from 7.6 million Twenty-foot Equivalent Unit (TEU)s in 2008 to 16.2
million TEUs in 2020. TEU is a unit of cargo capacity.

 36 www.visionias.in ©Vision IAS


.
 Q 85.B
x Dhuandhar Falls:
o The Dhuandhar Falls is located on the Narmada River in Bhedaghat and are 30 meters high. The
 Narmada River, making its way through the world-famous Marble Rocks, narrows down and then
plunges into a waterfall known as Dhuandhar. The plunge, which creates a bouncing mass of mist, is
so powerful that its roar can be heard from a far distance. The white marble rocks appear spectacular
when the moonlight falls on them, rendering them a silvery appearance. Hence pair 1 is correctly
matched.
x Shivanasamudra Falls:
o The Shivanasamudra Falls is on the Kaveri River after the river has found its way through the rocks
and ravines of the Deccan Plateau and drops off to form waterfalls. The island town of
Shivanasamudra divides the river into twin waterfalls. This creates the fourth-largest island in the
course of the river.
o This is a segmented waterfall. Segmented waterfalls occur where the water flow is broken into two or
more channels before dropping over a cliff, resulting in multiple sides by side waterfalls. It has an
average width of 305 metres and a height of 98 m. It is a perennial waterfall. Hence pair 2 is correctly
matched.
x Kanchenjunga Waterfalls:
o At a distance of 10 km from Yuksom and 24 km from Pelling, Kanchenjunga Waterfalls is a stunning
waterfall situated in the beautiful hill town of Pelling, Sikkim. It is one of the largest waterfalls in
Sikkim and among the prime Pelling Tourist Places.
o Kanchenjunga Falls is a perennial waterfall and it is believed that the pristine waterfall originates
high up in the glaciers of Mount Kanchenjunga, the 3rd highest peak in the world. It gushes down
from a height of 100 feet into a pool. Kanchenjunga falls was not known to people and remained
untouched till the 90s. It only came into the limelight, when a local tour operator named Topjor Bhutia
accidentally discovered it. Today this spot become one of the leading tourist spots in West
Sikkim. Hence pair 3 is not correctly matched.

Q 86.C
x Tamar Revolts (1789-1832) The tribals of Tamar revolted over 7 times between 1789-1832 against the
British. They were joined in the revolt by the tribals of adjoining areas - Midnapur, Koelpur, Dhadha,
Chatshila, Jalda and Silli. They revolted against ' the faulty align system of the government. The Tamar
revolts were led by Bhola Nath Sahay of Tamar. In 1832 the arrows of war circulated throughout the
region. Oraons, Mundas, Hos or Kols, who had distinct social and cultural identity joined the insurgents
under the leadership of Ganga Narain Singh, a member of Banbhum Raj family. Hence pair 1 is correctly
matched.
x Khond Uprisings (1837-1856) From 1837 to 1856, the Khonds of the hilly tracts extending from Odisha
to the Srikakulam and Visakhapatnam districts of Andhra Pradesh revolted against Company rule. Chakra
Bisnoi, a young raja, led the Khonds who were joined by the Ghumsar, Kalahandi and other tribals
to oppose the suppression of human sacrifice, new taxes, and the entry of zamindars into their
areas. With Chakra Bisnoi’s disappearance, the uprising came to an end. Hence pair 2 is correctly
matched.
x Bhuyan and Juang Rebellions by the Bhuyans, Juangs and Kals; first uprising (1867-68) was led by
Ratna Nayak; second uprising(1891-93) was led by Dharni Dhar Nayak (Kheonjhar, Orissa); against the
installation of a British protege on the throne after the death of their raja in 1867. Hence pair 3 is
correctly matched.

Q 87.A
x Warren Hastings tried to build up a framework of justice after the Mughal model. In 1772, a Divani Adalat
and a Faujdari Adalat were set up at the district level.
x The Diwani Adalat was presided over by the Collector who was competent to decide all civil cases
including those concerning personal property, inheritance, caste, marriage, debts etc. In case of Hindus, the
Hindu law was applicable, in case of Mastims the Muslim law. The Diwani Adalat could decide cases
involving sums up to Rs. 500 adove which appeals lay to the Sadar Diwani Adalat at Calcutta presided over
by the President and two members of the Supreme Council assisted by Indian officers.
x The District Faujdari Adalat was presided over by Indian officers of the Company who decided cases
with the assistance of Qazis and Muftis. The Collector, a European officer, was authorised to exercise
some control and supervision over the Faujdari Adalat (i.e. to see that the evidence was duly submitted and

 37 www.visionias.in ©Vision IAS


.
 weighed and the verdict passed was fair and impartial and given in open court). Hence statement 1 is
correct.
o The Mohammadan law was followed in the Faujdari Adalat. This Adalat could not award death
 sentence or order confiscation of property for which the confirmation of the Sad Nizamat Adalat was
necessary. Hence statement 2 is correct.
o Appeals from the Faujdari Adalat lay to the Sad Nizamat Adalat presided over by the Deputy
Nazim assisted by the Chief Qazi and the Chief Mufti and three Maulvis. The President and Council
supervised the proceedings of this Court. Hence statement 3 is not correct.

Q 88.A
x National E-Governance Service delivery Assesment (NeSDA) is an ongoing initiative by the
Government to assess the quality and effectiveness of e-governance service delivery across various
government departments and agencies. The initiative is aimed at promoting good governance practices
and improving the overall citizen-centric service delivery experience.
x NeSDA is a biennial study assesses States, Union Territories (UTs), and focus Central Ministries on the
effectiveness of e-Governance. Hence statement 1 is correct.
x NeSDA initiative has been undertaken by Department of Administrative Reforms & Public
Grievances (DAPRG), Govt. of India. Hence statement 2 is not correct.
x NeSDA has followed the Good Governance Index 2021 grouping of the States and UTs.
o North-East and Hill States make up the first group while Union Territories make up the second group.
o The remaining states of India have been classified into two states as Remaining States – Group A and
Remaining States – Group B.
x As per NeSDA findings of 2021:
o Among the North-East and Hill States, Meghalaya and Nagaland are the leading State Portals with
an overall compliance of more than 90% across all assessment parameters.
o Among Union Territories, Jammu & Kashmir ranked the highest with an overall compliance of
nearly 90%.
o Among the Remaining States, Kerala, Odisha, Tamil Nadu, Punjab, Karnataka and Uttar
Pradesh had a compliance of more than 85%.
o Amongst all the States and UTs, Kerala had the highest overall compliance score.
o Among the Services Portals for North-East and Hill States, the highest-ranking states of Meghalaya
and Tripura showed improvement across all six sectors compared to NeSDA2019.
o In the Union Territories category, Jammu & Kashmir was assessed for the first time in NeSDA
2021 and scored the highest amongst all UTs for six sectors.
o Among the Remaining States, the overall score of Tamil Nadu increased the most in 2021 compared
to 2019.
o Andhra Pradesh, Kerala, Punjab, Goa, and Odisha also improved the compliance of their Services
Portals by 100%.
o Punjab, Tamil Nadu, and Rajasthan are the leading states with compliance of more than 75% across all
parameters for their Services Portals.
o Among the focussed Central Ministries, Home Affairs, Rural Development, Education, and
Environment, Forest & Climate Change are the leading Ministry Portals with an overall compliance
of more than 80% across all assessment parameters.
o Ministry Portal of Home Affairs had the highest overall compliance score.
o The Central Public Procurement Portal, Digital Police Portal, and Bhavishya Portal are the leading
Ministry Services Portals with an overall compliance of more than 85% across all assessment
parameters.

Q 89.D
x Salinity is defined as the ratio between the weight of the dissolved materials and the weight of the sample
sea water. Generally, salinity is defined as ‘the total amount of solid material in grams contained in one
kilogram of seawater and is expressed as part per thousand.
x The oceanic salinity not only affects the marine organisms and plant community but it also affects the
physical properties of the oceans such as temperature, density, pressure, waves, and currents, etc.
o The ocean salinity has significant effects on the physical properties of seawater and other aspects
of the oceans as follows:
o The freezing and boiling points are greatly affected and controlled by the addition or subtraction of
salts in seawater. The saline water freezes slowly in comparison to fresh water. It is known to all that
pure water freezes at the temperature of 00C freezing point. If the salinity of seawater becomes 35%
 38 www.visionias.in ©Vision IAS
.
 then it would freeze at the temperature of – 1.910C. On the other hand, the boiling point of saline water
(seawater) is higher than fresh water. Hence options 1 and 4 are correct.
o Salinity and density of seawater are positively correlated i.e. the salinity of seawater increases its
 density because solutes (here salts) in water have greater atomic weight than the molecules of fresh
water. This is why man is seldom drowned in seawater with very high salinity.
o Evaporation is controlled by the salinity of the oceans. In fact, solutes (salts) in water lowers the rate
of evaporation in the oceans. Thus more saline water is less evaporated than less saline water. It may
be mentioned that evaporation also controls the salinity of seawater. More evaporation reduces the
volume of seawater and hence the concentration of salts increases (i.e., seawater salinity
increases). Hence options 2 is correct.
o Spatial variation in seawater salinity becomes a potent factor in the origin of ocean currents. Hence
options 3 is correct.
o The ocean salinity affects the marine organisms and plant community.

Q 90.B
x The Amaravati school of art developed under the patronage of dynasties like the Satavahanas and
Ikshvakus. Both bas-reliefs and free-standing sculptures were produced.
x Satavahana reliefs have been found at Amaravati, where they were used for decorating the stupa. They show
the stupa, Jataka tales, scenes from the life of the Buddha, and Buddhist symbols like chakra. Compared to
the Sunga specimens, there was a great improvement in technique.
x The reliefs created under the Ikshvakus show even further improvement. These have been recovered from
their capital, Nagarjunkonda. They were, again, used for decorating stupas. Great advancement is seen in
the usage of space. The figures are well-placed, have a slim appearance and look very lively. Hence
statement 1 is correct.
x Free-standing sculptures emerged towards the end of Satavahanas' reign, mainly of the Buddha
and Bodhisattvas, and have been discovered from Goli, Amaravati, and Nagarjunakonda. The material used
is marble or limestone.
x An important characteristic of the Amaravati school is the ‘narrative art’. The medallions were carved in
such a manner that they depict an incident in a natural way. For example one medallion depicts a whole
story of ‘taming of an elephant by the Buddha’. There is prominence of human figures rather than to figures
drawn from nature. Hence, statement 2 is correct.
x Although the school is deeply influenced by Mathura, it has certain distinctive features. The Buddha
is shown wearing a toga-like garment that only covers the left shoulder. His face is closely modelled on the
Buddha found on the reliefs.
x There is, however, a difference in the delineation of the body. Instead of a slimmer physique in the reliefs,
he has a massive body. His hair is in the form of short ring-like curls. Images of bodhisattvas, unlike in the
Mathura and Gandhara schools, are much rarer in the Amravati school. Hence statement 3 is not correct.

Q 91.C
x The Union Cabinet, chaired by the Hon’ble Prime Minister has approved setting up and promoting a
National Level multistate cooperative export society under the Multi-State Cooperative Societies
(MSCS) Act, 2002 with support from relevant Ministries especially the Ministry of External Affairs and
Department of Commerce, Ministry of Commerce and Industry through their export-related policies,
schemes & agencies by following the ‘Whole of Government Approach’ for undertaking exports of all
goods and services produced by cooperatives and related entities. Hence statement 1 is correct.
x Leading cooperatives — Indian Farmers Fertiliser Cooperative Limited (IFFCO), Krishak Bharati
Cooperative Limited (KRIBHCO), National Agricultural Cooperative Marketing Federation of India
(NAFED), Gujrat Cooperative Milk Marketing Federation (GCMMF), better known as Amul and
National Cooperative Development Corporation (NCDC) will be the promoters of the Society and
contribute ₹100 crore each. The society will have an authorized share capital of ₹2,000 crore with an area
of operation all over the country. It will have its registered office in Delhi. Hence statement 2 is not
correct.
x The Society will be different from the Export Promotion Council under the Ministry of Commerce which
only acts as a facilitator and provides information about the potential markets that can be tapped for a
particular product. This Society will provide end-to-end services to the cooperatives for undertaking
exports of all goods and services produced by cooperatives. It will open foreign bank accounts and
complete all the formalities, including necessary permissions for exporting a product. The icing on the cake
is that dividends will be shared with the manufacturer instantly and without any brokerage fee. Hence
statement 3 is correct.
 39 www.visionias.in ©Vision IAS
.
 x The Society will benefit the smallest of farmer or artisan who has a good product but does not have access
to the right platform. Through this Society, they will get access to international market and good returns
too. Once the product has been tested for international standards, the packaging and export will be done by
 the Society.

Q 92.A
x A 'bank run' occurs when a large number of customers of a bank or another financial institution withdraw
their deposits simultaneously due to concerns about the bank's solvency. As more people withdraw their
funds, the probability of default increases, thereby prompting more people to withdraw their deposits. In
extreme cases, the bank's reserves may not be sufficient to cover the withdrawals.
x When a bank cannot satisfy customer demands for withdrawals—or if there’s a rumor that the bank will be
unable to do so—the situation worsens. Customers fear being the “last one to the exit,” and they attempt to
withdraw as much as possible. In a worst-case scenario, a bank may be unable to meet obligations, leading
to complete failure.
x Hence option (a) is the correct answer.

Q 93.C
x Recently, Defence Research and Development Organisation (DRDO) successfully flight tested SFDR
booster.
x About Solid Fuel Ducted Ramjet (SFDR):
o It is a missile propulsion system that includes a thrust-modulated ducted rocket with a reduced smoke
nozzle-less missile booster.
o The first flight of SFDR, developed under a joint Indo-Russian R&D project, was tested in 2018. It had
achieved the speed of Mach 3.
o The system utilizes a solid fuelled airbreathing Ramjet Engine. Unlike solid-propellant rockets,
the Ramjet takes up oxygen from the atmosphere during flight. Thus, it is light in weight and can
carry more fuel. Hence statement 1 is correct.
o Range: Air targets in the ranges from 70 - 340 km.

x Hence statement 2 is correct and 3 is not correct.

Q 94.B
x While catching a fast-moving cricket ball, a fielder on the ground gradually pulls his hands backward with
the moving ball. In doing so, the fielder increases the time during which the high velocity of the moving
ball decreases to zero. Thus, the acceleration of the ball is decreased and therefore the impact of
catching the fast-moving ball is also reduced. If the ball is stopped suddenly, then its high velocity
decreases to zero in a very short interval of time. Thus, the rate of change of momentum of the ball will
be large. Therefore, a large force would have to be applied for holding the catch that may hurt the palm of
the fielder.
x Hence option (b) is the correct answer.

Q 95.C
x The Association of Asian Election Authorities (AAEA) is an international organization that aims
to promote and enhance the credibility, transparency, and efficiency of elections in Asia. The AAEA
comprises election management bodies and organizations from across Asia, including electoral
commissions, ministries of interior, and research institutes.
x India has been unanimously elected as the new Chair of the Association of Asian Election Authorities
(AAEA) for 2022-2024 at the recently held meeting of the Executive board and General Assembly at
 40 www.visionias.in ©Vision IAS
.
 Manila, Philippines in 2022. Commission on Elections, Manila was the current chair of AAEA. Hence
statement 1 is correct.
x The mission of the Association of Asian Election Authorities is to provide a non-partisan forum in the
 Asian region for sharing experiences and best practices among election authorities to discuss and act
upon ways to promote open and transparent elections with the objective of supporting good governance and
democracy.
x The Association of Asian Election Authorities (AAEA) was established in 1998. Currently 20 Asian
EMBs are members of AAEA. Election Commission of India is a founder member EMB of the AAEA
and also served on the Executive Board of the AAEA as the Vice Chair during 2011-13 and Chair during
2014-16. Hence statement 2 is correct.

Q 96.C
x Millet is a type of cereal that is a part of the grass family Poaceae. This small round whole grain is grown
in India and Nigeria, especially in Asia and Africa. Just like brown rice and quinoa, millet food can be
cooked for easy digestion.
x There are multiple types of millets. However, the most common varieties include Finger Millet, Foxtail
Millet, Pearl Millet, Proso Millet, Little Millet and Sorghum Millet.
o Sorghum Millet (Jowar) It is commonly called Jowar in India. The largest sorghum-producing states
in India are Maharashtra and Karnataka. Some varieties of Sorghum are used for ethanol production.
o Proso Millet (Chena / Barri) Also called broom corn millet, this grain is mostly found in the drier
regions of Asia, Australia, Africa, Europe, and North America. You do not need a lot of water to grow
this crop.
o Pearl Millet (Bajra) In India, you will find this crop by the name Bajra and is mostly grown in states
such as Gujarat, Rajasthan, Maharashtra, Uttar Pradesh, and Haryana.
o Foxtail Millet (Kakum / Kangni) Foxtail millet or Italian millet is usually grown in a semi-arid region.
The growing season of this type of millet in India is very short.
o Finger Millet (Ragi)The common name for finger millet in India is ragi. The ragi millet is popular for
its nutritional benefits. It is one of the high-grown millets in India and ragi flakes are mostly used in
baby foods.
o Browntop Millet (Korle)This millets type is mostly grown in Karnataka and Andhra Pradesh. What
makes this millet unique is that it can be grown even in less fertile soil.
o Barnyard Millet (Sanwa)This is one of the millets in India that is grown in certain parts of Andhra
Pradesh and Uttarakhand. Its specialty is that it grows even in marginal lands.
o Little Millet (Moraiyo) Whether there is water logging or drought, this millet can be easily grown.
This type of millet is usually found in the Eastern Ghats in India.
o Buckwheat Millet (Kuttu) Buckwheat is one of the most popular varieties of millet, it is also known
as kuttu in India and is frequently consumed during the Navratra fasting period. It lowers blood pressure
and is diabetic-friendly. It is beneficial for cardiovascular health, and you should include it in your diet
if you wish to lose weight. Additionally, buckwheat safeguards against gallstones, asthma in children,
and breast cancer.
o Amaranth Millet (Rajgira) Amarnath also known as Rajgira, is a great source of protein and dietary
fiber. It is excellent for a balanced diet. Additionally, this millet aids in preventing hair loss and greying.
Amaranth also reduces cholesterol and the chance of developing cardiovascular disease. It contains a
lot of calcium, vitamins, and other nutrients.
o Kodo Millet Kodo millet is a digestible kind of millet that contains more of the amino acid lecithin. It
helps to strengthen the neurological system. Niacin, B6, and folic acid, among other B vitamins and
other vitamins and minerals, are especially abundant in kodo. It has minerals such as calcium, iron,
potassium, magnesium, and zinc. It is excellent for people who are gluten intolerant because it is a
gluten-free millet. When used consistently by postmenopausal women, it can treat cardiovascular
problems like excessive blood pressure and cholesterol levels.
x Hence option (c) is the correct answer.

Q 97.B
x Jal Jeevan Mission (JJM) is envisioned to provide safe and adequate drinking water through individual
household tap connections by 2024 to all households in rural India. Hence, statement 1 is correct.
x For ensuring long-term availability of the same, the program will also implement source sustainability
measures as mandatory elements, such as recharge and reuse through greywater management, water
conservation, and rainwater harvesting.

 41 www.visionias.in ©Vision IAS


.
 x The Jal Jeevan Mission will be based on a community approach to water and will include extensive
Information, Education, and communication as key components of the mission.
x Once a village is declared as Har Ghar Jal, the Gram Panchayat of that village conducts a special Gram
 Sabha and passes a resolution with the concurrence of all the village members that all the households,
schools, Anganwadi, and public institutions in their village have functional tap connection and
thus declare themselves as 'Har Ghar Jal Certified. Hence, statement 2 is not correct.
x Goa become the first ‘Har Ghar Jal’ certified State whereas Dadra & Nagar Haveli and Daman &
Diu (D&NH and D&D) become the first ‘Har Ghar Jal’ certified UT in the country. All the villages
within these have declared their village as ‘Har Ghar Jal’ through a resolution passed by Gram Sabha,
certifying that all households in the villages have access to safe drinking water through taps, ensuring that
‘No One is Left Out’. Hence statement 3 is correct.

Q 98.D
x National Livestock Mission is an initiative of the Ministry of Agriculture and Farmers’ Welfare. The
mission, which commenced in 2014-15, has the objective of sustainable development of the livestock sector.
x The list of beneficiaries includes:
o Farmers
o Individual entrepreneurs
o NGOs
o Companies
o Cooperative Groups of organized and unorganized sectors which include
ƒ Self- Help Groups (SHGs) and
ƒ Joint Liability Groups (JLGs). Hence option (d) is the correct answer.

Q 99.C
x Society of Integrated Coastal Management (SICOM), The Ministry of Environment, Forest and
Climate Change (MoEFCC) in pursuit of promoting its policies for sustainable development in coastal
regions have embarked upon a highly acclaimed program “BEAMS” (Beach Environment & Aesthetics
Management Services) under its ICZM (Integrated Coastal Zone Management) project. Hence
statement 2 is correct.
o The objective of the BEAMS program is to abate pollution in coastal waters, promote sustainable
development of beach facilities, protect & conserve coastal ecosystems & natural resources, and
seriously challenge local authorities & stakeholders to strive and maintain high standards of cleanliness,
hygiene & safety for beachgoers in accordance with coastal environment & regulations. This program
promotes beach recreation in absolute harmony with nature.
o This is one of the several other projects of ICZM that the Government of India is undertaking for the
sustainable development of coastal regions, striving for globally recognized and the coveted eco-
label ‘Blue flag” awarded by The Foundation for Environment Education (FEE) Denmark. Hence
statement 1 is correct.
x About FEE
o The FEE in Denmark awards the Blue Flag certification, which is a worldwide eco-level tag. At all
times, FEE Denmark monitors and audits for strict adherence to the 33 criteria. A beach, marina, or
sustainable boating tourism operator receives the certification. It functions as an environmental label.
It is given only once a year.
o The United Nations Environment Programme (UNEP), the United Nations World Tourism
Organisation (UNWTO), the Danish NGO Foundation for Environmental Education (FEE), and
the International Union for Conservation of Nature (IUCN) form the jury that awards the “Blue
Flag” label.

Q 100.A
x The Congress had organised a National Conference on Education in 1937 in Wardha. In the light of
the resolutions passed there, Zakir Hussain Committee formulated a detailed national scheme for basic
education. The main principle behind this Wardha scheme of basic education was 'learning through
activity'. Hence statement 2 is correct.
x It was based on Gandhiji's ideas published in a series of articles in the weekly 'Harijan'. Gandhiji
thought that western education had created a gulf between the educated few and the masses and had also
made the educated elite ineffective. Hence statement 1 is correct.
x Following Wardha conference, a committee under Dr Zakir Hussain was appointed to formulate the scheme
of the basic education. The aim of the basic education was to develop the qualities of the ideal citizenship
 42 www.visionias.in ©Vision IAS
.
 and more aspect should be given to the Indian culture than the literacy. Also, - There was no place for
English in the curriculum. There was no place for religious education in this scheme. The scheme centred
around ‘ manual productive work’ which might cover the remuneration of the teachers. It envisaged a seven
 year course through the mother tongue of the students.
x There was not much development on the ideas of basic education laid down by Wardha scheme
because of the start of the Second World War and the resignation of Congress ministries in 1939.
Hence, statement 3 is not correct.

 43 www.visionias.in ©Vision IAS

You might also like